You are on page 1of 57

DETAILED ANSWER KEY – ENVIRONMENT AND ECOLOGY

1. B
Approach
● Statement 1 is a general statement and hence can be assumed as correct.

Statement 1 >> Correct


● Genes can be introduced, enhanced or deleted within a species, across species or even
across kingdoms.

Statement 2 >> Incorrect


● No GM food crop has been released for Commercial cultivation.
● Only genetically modified cash crop for cultivation is BT Cotton

Statement 3 >> Correct


● Food Safety and Standards Authority of India (FSSAI) is the competent authority to
regulate GM foods through the inclusion of “genetically modified or engineered food or
food containing such ingredients” within the definition of food.
Additional information
● For Genetically Engineered organisms Genetic Engineering Appraisal Committee is the
apex body.

2. C
Approach
● Aspirants are advised to learn about the different zones of a lake/pond.

Statement 1 >> Correct


● Littoral zone is the shallow water zone of the lake which supports rooted vegetation.

Statement 2 >> Correct


● Profundal zone is the zone which does not receive any light.

Additional Information
● Limnetic zone is the ideal location for the growth of phytoplankton.
● Benthic zone is the deepest layer among all the zones of lakes/water bodies, they are
habitat for benthic organisms like snail etc.

3. C
Approach
● If you have a basic idea about virus, you can easily eliminate ‘virus’ from the list of
decomposers.

Statement 3 >> Incorrect

1
iPTS24 ENVIRONMENT ilearnoffc@gmail.com | 8089166792
● Virus - not a decomposer organism

Additional Information
● The organisms that carry out the process of decay or breakdown of the dead organism are
known as decomposers.
● Most decomposers are microscopic organisms, including protozoa and bacteria.
● Decomposers include fungi along with invertebrate organisms sometimes called
detritivores, which include earthworms, termites, and millipedes.

4. A
Approach
● Tropical rainforests get heavy rainfall. Thus, one can assume that soil will be deficient
in nutrients due to leaching.
● When we get Statement 2 as correct, Statement 1 can also be assumed as correct by
using common sense and logic.

Statement 1 >> Correct


● The regeneration of a tropical rainforest is not as rapid as that of a tropical deciduous
forest when it is removed.

Statement 2 >> Correct


● The soils of the tropical rain forest are typically nutrient-poor.
● Any nutrients in the soil would be swiftly leached away by the heavy rainfall.
● Thus, it does not regenerate quickly as compared to a tropical deciduous forest.
● Also is the correct explanation of Statement 1.

5. B
Approach
● This is a factual question.

Option B >> Correct


● Bergmann's Rule declares that temperature also affects the absolute size of an animal
and the relative proportions of various body parts. Birds and mammals attain greater
body size in cold regions than in warm areas.

Additional Information
● Allen's Rule expresses that the tail, snout, ears and legs of mammals are relatively shorter
in colder areas than in the warmer areas. Allen's rule says that endotherms (Birds and
Mammals) from low temperature or colder climatic conditions generally have shorter
appendages or limbs than the alike animals from higher temperature or warm climatic
conditions.
● Cope's Rule states that the lineages of the population tend to increase in body size over
evolutionary time i.e bodies get larger over time i.e population genealogies will in general
increment in body size over evolutionary time.
● Gloger' Rule states that some insects, birds and mammals in warm humid climates bear
darker pigment than the races of the same species present in cool and dry climates.

2
iPTS24 ENVIRONMENT ilearnoffc@gmail.com | 8089166792
6. C
Approach
● Here, one can take hints from words ‘overshoot’, ‘deficit’ and ‘reserve’.
● By correlating the words with the description provided, one can easily assume that all
three statements are correct.

Statement 1 >> Correct


● Ecological overshoot is the phenomenon which occurs when the demands made on a
natural ecosystem exceed its regenerative capacity.

Statement 2 >> Correct


● An ecological deficit occurs when the Footprint of a population exceeds the bio capacity
of the area available to that population.

Statement 3 >> Correct


● An ecological reserve exists when the biocapacity of a region exceeds its population's
Footprint.

7. A
Approach
● If you have a basic idea that fish droppings contain phosphate, you can take a guess
that Both Statement 1 and 2 are correct and Statement 2 is the correct explanation of
Statement 1.

Statement 1 >> Correct


● Marine birds play a unique role in the phosphorus cycle.

Statement 2 >> Correct


● Marine birds take up phosphorus from ocean fish.
● Their droppings on land (guano) contain high levels of phosphorus and are sometimes
mined for commercial use.
● A 2020 study estimated that the ecosystem services (natural processes and products that
benefit humans) provided by guano are worth $470 million per year.

8. D
Approach
● Statements 1, 2 and 4 can be assumed as correct from basic common sense.
● For tackling Statement 3, you need to have knowledge about Black-band disease.

Statement 1 >> Correct


● Sedimentation has been identified as a primary stressor for the existence and recovery
of coral species and their habitats.
● Sediment deposited onto reefs can smother corals and interfere with their ability to feed,
grow, and reproduce.

Statement 2 >> Correct


● Overfishing can alter food-web structure and cause cascading effects, such as reducing
the numbers of grazing fish that keep corals clean of algal overgrowth. Blast fishing (i.e.,
using explosives to kill fish) can cause physical damage to corals as well.
3
iPTS24 ENVIRONMENT ilearnoffc@gmail.com | 8089166792
Statement 3 >> Correct
● Black band disease (BBD) is an infectious coral disease that is globally present and
affects 19 coral species.
● Black band disease is characterized by a tissue loss lesion demarcated by a black band,
which spreads across a coral colony ultimately resulting in its death.

Statement 4 >> Correct


● Degraded plastics and micro plastics (e.g., beads in soap) can be consumed by coral,
fish, sea turtles, and other reef animals, blocking their digestive tracts and potentially
introducing toxics.

9. A
Approach
● Here, one needs to know about the word meaning of lentic and lotic.
● Then, we can arrive at the answer by analysing the nature of given water bodies.

Statement 1 and 4>> Correct


● Ponds and Bogs are included in the lentic freshwater ecosystems.

Statement 2, 3 and 5 >> Incorrect


● Springs, Brooks and Creeks are included in the lotic freshwater ecosystems.

Additional Information
● Freshwater habitats can be further divided into two groups as lentic and lotic
ecosystems based on the difference in the water residence time and the flow velocity.
● The term lentic (meaning ‘to make calm') is used for still waters, which offer
environmental conditions, which differ sharply with that of the streams. Light penetrates
only to a certain depth depending upon turbidity.
● Water bodies such as lakes, ponds, pools, bogs, and other reservoirs are standing water
and known as lentic habitats.
● A bog or bogland is a wetland that accumulates peat as a deposit of dead plant materials
– often mosses, typically sphagnum moss.
● The term lotic (from lavo, meaning ‘to wash') represents running water, where the entire
body of water moves in a definite direction.
● These may comprise brooks, streams, rivers, creeks and springs.
● Brook is a term used for the small body of water while river is a term used for a relatively
large natural body of water.
● The stream is generally designated as smaller than a river but bigger than a brook.
● Spring is an issue of water from the earth, which takes the form of a stream on the surface

10. A
Approach
● In order to remove a pollutant from atmosphere, something like a scrub will serve the
purpose. Hence Statement 2 explains statement 1. This would be a logical assumption if
we don't know about hydroxyl radicals.

Statement 1 >> Correct


4
iPTS24 ENVIRONMENT ilearnoffc@gmail.com | 8089166792
● Methane is removed from the atmosphere by oxidation of methane by hydroxyl radical.
After reaction methane is converted to Carbon dioxide and Water.

Statement 2 >> Correct


● Hydroxyl radicals are a form of sink because they scrub the atmosphere clean of
pollutants and break them down. For this reason they are called cleanser of the
atmosphere.
● Also Statement 2 explains statement1.

Additional information
● Methane is a more potent Greenhouse Gas than Carbon dioxide.
● However it is a short Lived Climate Pollutant.
Why this question?
Biogeographic cycles are important for UPSC. 2 questions were asked
previously.

11. D
Approach
● Aspirants are advised to have a clear understanding of basic terminologies in ecology
like ‘Ecological Niche’.

Statement 1 >> Correct


● Ecological niche is a term for the position of a species within an ecosystem, describing
both the range of conditions necessary for persistence of the species, and its ecological
role in the ecosystem.
● Ecological niche subsumes all of the interactions/interrelationship between a species
and the biotic and abiotic environment, thus represents a very basic and fundamental
ecological concept.

Statement 2 >> Correct


● Theoretically, if two species have the same niche, one species will exclude the other. The
corollary is that the niches of coexisting species must differ.
● Niches of coexisting species can be similar, but not identical.
● Thus, the number of niches in an environment can determine the number of species
present or the variety of the ecosystem.

Statement 3 >> Correct


● Ecological niches are found in all types of ecosystems.
● The presence of ecological niches is a fundamental concept in ecology, highlighting the
dynamic and interconnected nature of ecosystems, regardless of whether they are
terrestrial, aquatic, tropical, temperate, or any other type.

12. B
Approach
● From the basic knowledge that mangroves are mainly found in the coastal areas, one
can assume that they will be having remarkable salt water tolerance.

5
iPTS24 ENVIRONMENT ilearnoffc@gmail.com | 8089166792
Statement 1 >> Correct
● It produces pneumatophores (blind roots) to overcome respiratory problems in the
anaerobic soil conditions.

Statement 2 >> Correct


● Mangroves exhibit Viviparity mode of reproduction. i.e. seeds germinate in the tree itself
(before falling to the ground).
● This is an adaptive mechanism to overcome the problem of germination in saline water.

Statement 3 >> Incorrect


● Mangroves are trees and bushes growing below the high water level of spring tides which
exhibits remarkable capacity for salt water tolerance.

13. C
Approach
This is a factual question. Attempt this type of questions only with factual clarity.

Statement 1 >> Incorrect


● The 'Global Framework on Chemicals – For a planet free of harm from chemicals and
waste' was formally established at the fifth International Conference on Chemicals
Management (ICCM5) in Bonn, Germany, held on September 25-29, 2023.

Statement 2 >> Correct


● The framework calls for, by 2035, a phase out of highly hazardous pesticides in
agriculture where the risks have not been managed and safer alternatives are available.

Statement 3 >> Correct


● A decision was made to unlock financing for the implementation of the framework from
different sources. A UNEP-administered Global Framework on Chemicals Fund will be
set up, time-limited, that may include multilateral, bilateral and private sector sources.
Why this question?
Following an all-night negotiating session and years of talks, the fifth
International Conference on Chemicals Management (ICCM5) in 2023
September formally adopted a new global framework for the integrated
management of chemicals and waste beyond 2020, the ‘Global
Framework on Chemicals – For a planet free of harm from chemicals and
waste.’

14. D
Approach
● Here, one can recollect the facts by studying that ‘Auto’ means ‘within’ and ‘Allo” means
‘outside’

Statement 1 >> Incorrect


● When succession is brought about by living inhabitants of that community itself, the
process is called autogenic succession,

6
iPTS24 ENVIRONMENT ilearnoffc@gmail.com | 8089166792
Statement 2 >> Incorrect
● When succession is brought about by outside forces is known as allogenic succession.

15. C
Approach
● Statement 1 and 2 are basic facts regarding corals and can be assumed as correct.
● For tackling Statement 3, you need to know about the difference between soft corals and
hard corals.

Statement 1 >> Correct


● Corals have a symbiotic relationship with microscopic algae called zooxanthellae that
live in their tissues.
● The corals receive their coloration from the zooxanthellae.

Statement 2 >> Correct


● A coral's prey is typically microscopic zooplankton.
● During feeding, a coral polyp will extend its tentacles out from its body and wave them in
the water current where they encounter zooplankton, bacterioplankton, or other food
particles.

Statement 3 >> Correct


● Soft coral, also known as Alcyonacea and ahermatypic coral, do not produce a rigid
calcium carbonate skeleton and do not form reefs, though they are present in reef
ecosystems.

Additional Information
● Hard corals, also known as scleractinian and stony coral, produce a rigid skeleton made
of calcium carbonate (CaCO3) in crystal form called aragonite.
● Hard corals are the primary reef-building corals. Colonial hard corals, consisting of
hundreds to hundreds of thousands of individual polyps, are cemented together by the
calcium carbonate 'skeletons' they secrete.
● Hard corals that form reefs are called hermatypic corals.

16. B
Approach
● To solve this question, you only need to have a basic idea about Tundra biome.
● Option A, C and D can easily be associated with climatic conditions of Tundra biome. It
can be assumed as correct and thus can be eliminated.

Option A >> Correct


● Tundra insects have also developed adaptations for the cold;
● Mosquitoes (Aedes nigripes) for example, have a chemical compound that acts as
antifreeze, lowering the freezing temperature in their bodily fluids.

Option B >> Incorrect


● There are very few reptiles and amphibians found in the tundra because the
temperatures are so cold.
● Although amphibians and reptiles account for nearly 15,000 species worldwide, only five
amphibians and a single reptile are found in the Arctic.
7
iPTS24 ENVIRONMENT ilearnoffc@gmail.com | 8089166792
Option C >> Correct
● Animals in the tundra also adapted to extreme conditions.
● The animals are protected from chillness by the presence of thick cuticles and
epidermal hair or fur.

Option D >> Correct


● Tundra winters are long, dark, and cold, with mean temperatures below 0°C for six to 10
months of the year.
● The temperatures are so cold that there is a layer of permanently frozen ground below the
surface, called permafrost.
● This permafrost is a defining characteristic of the tundra biome. In the tundra
summers, the top layer of soil thaws only a few inches down, providing a growing surface
for the roots of vegetation.

Additional Information
● Tundra wildlife includes small mammals—such as Norway lemmings (Lemmus lemmus),
arctic hares (Lepis arcticus), and arctic ground squirrels (Spermophilus parryii)—and
large mammals, such as caribou (Rangifer tarandus).

17. D
Approach
● This question demands detailed knowledge about different types of interactions between
the organisms.

Pair 1 >> Correct


● Whales And Barnacles - example of Commensalism

Pair 2 >> Correct


● Penicillium and Bacteria - example of Amensalism

Pair 3 >> Correct


● Aphids and Ants - example of Mutualism

Pair 4 >> Correct


● Sea lamprey and Other fish species - example of Parasitism

Additional Information
● Barnacles, which belong to the class of crustaceans, go through a cycle that necessitates
a stationary phase.
● During their larval development, they seek out and adhere to surfaces that offer them a
chance at survival and growth.
● Marine mammals such as whales often become unwitting hosts to these organisms. As
the whales journey through the oceans, the barnacles affixed to their skin benefit from a
mobile existence that they could not achieve on their own.
● This arrangement allows barnacles access to a broader feeding range and a steady stream
of plankton-rich waters, courtesy of their host's movement through various marine
environments.
● Notably, the barnacles do not damage the whales because they do not consume the
whale's tissue or blood.

8
iPTS24 ENVIRONMENT ilearnoffc@gmail.com | 8089166792
● In the case of Penicillium (a type of fungus) and bacteria, Penicillium produces
antibiotics, like penicillin, which can inhibit the growth of bacteria in its vicinity. This
inhibitory effect harms the bacteria without any benefit to the fungus.
● Aphids are little sap-sucking insects that secrete honeydew, a sugary liquid that is the
waste product of their diet.
● Many aphid species are known to engage in a mutualistic relationship with ants that feed
on the honeydew by 'milking' the aphids with their antennae.
● In return, some species of ants will protect the aphids from predators and parasites. Some
will move aphid eggs and nymphs underground to their nest, which ultimately makes
harvesting their honeydew more efficient - like an ant equivalent of a dairy farm.
● Sea lamprey is a parasite that attaches itself to the body of a host fish and consumes the
fish's body fluids.
● The lamprey draws nourishment from the fish, and the fish gradually weakens over time.

18. D
Approach
● Although the question is fact-based, certain logics can be applied to reach the answer.
Statement 1 might be incorrect because it would be disastrous if the level of extraction is
far higher than recharge. Similarly, Statement 2 is also an extreme statement. And, you
might be knowing that northwest India is more prone to the issue than any other place in
India; making Statement 3 incorrect.

Statement 1 >> Incorrect


● As per the 2023 assessment report, the total annual ground water recharge for the
entire country is 449.08 billion cubic meters (BCM), marking an increase of 11.48
BCM compared to the previous year (2022) and annual ground water extraction for the
entire country is 241.34 BCM.

Statement 2 >> Incorrect


● Out of the total 6553 assessment units in the country, 736 units have been
categorized as ‘Over-exploited’ and 4793 units as ‘safe’.

9
iPTS24 ENVIRONMENT ilearnoffc@gmail.com | 8089166792
Statement 3 >> Incorrect

Why this question?


The Union Minister of Jal Shakti, Shri Gajendra Singh Shekhawat released
the Dynamic Ground Water Resource Assessment Report for the entire
country for the year 2023. The assessment was carried out jointly by the
Central Ground Water Board (CGWB) and States/UTs.

19. C
Approach
● If you know that Mangroves represent an ecotone between marine and terrestrial
ecosystems, Statement 2 can be considered as incorrect.
● Here, you can directly arrive at the answer by eliminating options A, B and D.

Statement 1 >> Correct


● Ecocline is a zone of gradual but continuous change between adjacent ecosystems, where
a sharp boundary cannot be demarcated in terms of species composition. It occurs across
the environmental gradient (i.e., change in abiotic factors such as salinity, temperature,
altitude, etc.)

Statement 2 >> Incorrect


● The Mangrove Forests represent an ecotone between Marine and Terrestrial ecosystems.
● Estuaries represent the ecotone between salwater and freshwater ecosystems

Statement 3 >> Correct


● The Grasslands represent an ecotone between desert and forest.
10
iPTS24 ENVIRONMENT ilearnoffc@gmail.com | 8089166792
20. A
Approach
● The concepts discussed in the question are covered in all standard textbooks.
● So, if you fail to solve this question, you are advised to revise the standard books
thoroughly.

Statement 1 >> Correct


● There are certain limitations to ecological pyramids, such as the fact that they do not
account for the same species belonging to two or more trophic levels.
● Furthermore, although playing an important role in the environment, saprophytes are
not given a place in ecological pyramids.

Statement 2 >> Incorrect


● In case of aquatic ecosystems, the pyramid of biomass may be inverted.
● For example, in a pond, phytoplankton are the main producers. They have very short
life cycles and a rapid turnover rate (i.e. they are rapidly replaced by new plants). Therefore
their total biomass at any given time is less than the biomass of herbivores supported by
them.

Additional Information:
● Saprophytes are the living organisms that live and feed on dead and decaying organisms.
They are considered extremely important in soil biology.

21. D
Approach
● This Statement 1 is incorrect. Biomass co-firing is typically employed to reduce the carbon
footprint by incorporating renewable biomass alongside traditional fossil fuels. A higher
carbon footprint is generally associated with greater environmental impact.

Statement 1 >> Incorrect and Statement 2 >> Correct


● Biomass co-firing is a process where biomass, such as wood pellets, agricultural
residues, or other organic materials, is combined with traditional fossil fuels, like
coal or natural gas, and burned together to produce energy.
● This approach is used in power plants and industrial facilities as a way to reduce
greenhouse gas emissions and promote more sustainable energy production.
● By blending renewable biomass with fossil fuels, co-firing can decrease the carbon
footprint of energy generation and make use of readily available organic materials.

Additional information
● It’s considered a transitional step towards cleaner and more environmentally friendly
energy production.
● Coal and biomass are combusted together in boilers that have been designed to burn coal.
For this purpose, the existing coal power plant has to be partly reconstructed and
retrofitted.
● The Ministry of Power has identified various surplus agro residues that can be utilised
for biomass pellet production. These include agro-residues from crops such as Paddy,
Soya, Arhar, Gwar, Cotton, Gram, Jawar, Bajra, Moong, Mustard, Sesame, Til, Maize,
Sunflower, Jute, Coffee, etc. and shell waste products like Groundnut Shell, Coconut
Shell, Castor Seed Shell, etc.
11
iPTS24 ENVIRONMENT ilearnoffc@gmail.com | 8089166792
● The Ministry of Power issued modification on 16.06. 2023 to revise the biomass policy
dated 08.10. 2021 and now it mandates 5% biomass co-firing in Thermal Power Plants
(TPPs) from FY 2024-25. This obligation shall increase to 7% from FY 2025-26
Why this question?
Biomass co-firing method is appearing in the news due to the start of the
stubble burning season in Punjab and Delhi air pollution.

22. A
Approach
● This is a factual as well as concept based question. Though a thorough understanding of
these concepts is needed, a wild guess may be made by correlating “border adjustment
mechanism” with something done for preventing a negative thing happening across
borders; reaching Option A.

● CBAM is a tariff system that aims to prevent carbon leakage by imposing charges
on imports based on their carbon content. It extends the concept of carbon pricing to
imported goods, ensuring that products from regions with less stringent carbon
regulations face a similar cost to those produced in regions with stricter regulations. This
mechanism encourages global partners to adopt more sustainable practices to avoid
tariffs.
● The European Union proposed CBAM as part of the European Green Deal in July 2019.
The European Union officially adopted CBAM in July 2021.
● Carbon leakage occurs when industries or activities move from regions with strict
carbon regulations to areas with more lenient regulations. This relocation can result
in increased carbon emissions globally, defeating the purpose of emission reduction
efforts in specific regions.

Additional Information
● Carbon neutrality, or net-zero emissions, is achieved when an entity (like a company or
a nation) balances the amount of greenhouse gases it emits with an equivalent
amount removed or offset. It involves reducing emissions and investing in projects that
absorb or offset an equivalent amount of carbon.
● Carbon offsetting involves compensating for emissions by investing in projects that
reduce or capture an equivalent amount of greenhouse gases. These projects can
include reforestation, renewable energy initiatives, or methane capture. The goal is to
achieve a balance between emitted and offset carbon.
● Carbon surplus occurs when the amount of carbon emissions released into the
atmosphere exceeds the capacity of natural systems or human activities to absorb
or offset them. This contributes to the overall increase in greenhouse gas concentrations,
leading to environmental concerns and climate change impacts.
Why this question?
Recently, the concerns of Indian steel exporters regarding the tariff
implications of the CBAM was widely reported.

23. A
Approach:
● Freshwater availability is one of the favourite areas of UPSC.
● If you fail to solve this question, you are advised to learn the basic NCERTs as the

12
iPTS24 ENVIRONMENT ilearnoffc@gmail.com | 8089166792
question is directly sourced from it.

Statement 1 >> Incorrect


● Approximately, 71% of the earth’s surface is covered with it but fresh water constitutes
only about 3% of the total water.

Statement 2 >> Correct


● India accounts for about 2.45% of the world's surface area, 4% of the world’s water
resources and about 16 % of world’s population.

Statement 3 >> Incorrect


● According to the United Nations' World Water Development Report, 2022, India extracts
more than a quarter of the world's groundwater withdrawal each year.

Additional Information
● 2.5% of the earths freshwater is unavailable: locked up in glaciers, polar ice caps,
atmosphere, and soil; highly polluted; or lies too far under the earth's surface to be
extracted at an affordable cost.
● Thus, on the planet Earth, the freshwater available for use amounts to about less than
1% of the total water found.
24. A
Approach
● Statement 3 is only logical as the Government is increasingly promoting the AYUSH
medicines in recent times. In the same pattern, you may find Statement 1 as illogical
because the Government is nowadays pushing for decriminalising many offences.

Statement 1 >> Incorrect


● In the recent Biological Diversity (Amendment) Act, 2023, penalties have been
revised under Section 55, shifting from imprisonment to increased financial
penalties.
● Section 55A introduces the appointment of an adjudicating officer to oversee inquiries and
impose penalties. This officer, along with authorities under Section 55B, has the authority
to conduct searches, seizures, and collect evidence. Section 61 now permits any individual
to file a complaint for potential infractions under the Act.

Statement 2 >> Incorrect


● The Forest (Conservation) Act was amended recently. The Amendment’s principal
thrust is that it redefines what a ‘forest’ is in Indian law. It stipulates that only those
lands that were notified as ‘forest’ under the Indian Forest Act 1927, any other
relevant law or were recorded as ‘forests’ in government records will be
acknowledged as ‘forests’ under the Act as well.

Statement 3 >> Correct


● The Act now recognises "codified traditional knowledge", ensuring protection of
established practices. For sectors like AYUSH, this means that traditional practices
documented in ancient texts are acknowledged and safeguarded, promoting their
authentic use and preventing misappropriation.

13
iPTS24 ENVIRONMENT ilearnoffc@gmail.com | 8089166792
● According to the amendment, “codified traditional knowledge” means the knowledge
derived from authoritative books specified in the First Schedule to the Drugs and
Cosmetics Act, 1940.
Why this question?
All the three statements are framed from the recent changes in the Acts.
Aspirants are advised to note the major changes in the Acts.

25. A
Approach:
● Word deconstruction and option elimination can be employed to arrive at the answer.
The given options has descriptions of Aeroponics, Polyculture and Permaculture.

Option A >> Correct


● Paludiculture is the productive land use of wet and rewetted peatlands that preserves
the peat soil and thereby minimizes CO2 emissions and subsidence. With paludiculture,
peatlands are kept productive under permanently wet, peat-conserving and potentially
peat-forming conditions. Rewetting (i.e. raising the water level near to surface) is
essential to minimize emissions and peat degradation, but also impedes drainage-based
land use.

26. C
Approach:
● This question can be solved by using simple common sense.
● Understanding the word ‘hypoxic’ is the key here.

Statement 1 >> Correct


● Dead zones are low-oxygen, or hypoxic, areas in the world’s oceans and lakes.

Statement 2 >> Correct


● Dead zones are caused by eutrophication, oil spills, dumping of toxic chemicals, and other
human activities.

27. A
Approach
● When you compare the words “green” and “carbon”, the former is a general term and the
latter is a specific term. Hence it might be safe to assume that both the statements are
correct and Statement 2 is the correct explanation of Statement 1.

Statement 1 and 2 >> Correct


● The Green Credits Programme, launched by the Environment Ministry in October this
year, is an effort to create a market-based incentive for different kinds of
environment-positive actions, not just for carbon emission reductions.
● Such a market-based system already exists for carbon, at the national as well as
international level, that allows trade in carbon credits. Companies, or nations, can claim
carbon credits if they take action to reduce their carbon footprint. These credits can be
traded for money. Companies unable to achieve their emission standards pay to buy these
credits and improve their performance.

14
iPTS24 ENVIRONMENT ilearnoffc@gmail.com | 8089166792
● The Green Credits programme attempts to replicate this mechanism for other
environmental actions, like water conservation or soil improvements. Methodologies
and standards to measure and verify such actions are still being developed. The market
would also need to be developed. As a starting point, it is envisaged that private companies
would buy these green credits as part of their CSR obligations.
● Unlike the carbon markets, which are more focused on industry and corporations,
green credit programme can benefit individuals and communities as well.
Why this question?
At COP28, PM Modi offered the concept to the international community,
in the hope to create a market for green credits at an international level, just
like the one on carbon credits exists.

28. B
Approach:
● Alfalfa and Amaranth were asked as options in UPSC PQ 2022.
● Strong grip in PYQ will help you to eliminate options C and D.

Statement 1 >> Correct


● Azotobacter - example of nitrogen fixing bacteria

Statement 2 >> Correct


● Bacillus - example of nitrogen fixing bacteria

Statement 3 >> Correct


● Clostridium - example of nitrogen fixing bacteria

Statement 4 >> Correct


● Klebsiella - example of nitrogen fixing bacteria

Statement 5 >> Incorrect


● Alfalfa - example of nitrogen fixing plant (not bacteria)
● Alfalfa is known as the “Queen of Forages” because of its high yield of high protein forage
with a relatively fast ruminal digestion rate.
● Alfalfa should also be dubbed the “King of Nitrogen Fixation” because alfalfa makes more
residual nitrogen available to subsequent crops than any other legume you can grow.

Statement 6 >> Incorrect


● Amaranth - not an example of nitrogen fixing bacteria
● Amaranthus, collectively known as amaranth, is a cosmopolitan genus of annual or short-
lived perennial plants.
● Some amaranth species are cultivated as leaf vegetables, pseudo cereals, and ornamental
plants.
● Most of the species from Amaranthus are summer annual weeds and are commonly
referred to as pigweed.
Additional Information
● Nitrogen-fixing bacteria are known to form symbiotic associations with some members
of all major groups of plants, as well as with some fungi.
● Although there are numerous reports of nitrogen-fixing bacteria occurring in animals, for
example termite guts, the significance to their hosts remains to be proven.
15
iPTS24 ENVIRONMENT ilearnoffc@gmail.com | 8089166792
● Nitrogen-fixing bacteria reduce the atmospheric nitrogen to ammonia and then assimilate
it to organic compounds such as amino acids. Thus, N₂-fixing bacteria play a key role in
global nitrogen cycles.
● Examples of nitrogen-fixing bacteria include species of Azotobacter, Bacillus,
Clostridium, and Klebsiella.

29. B
Approach
● This is a factual question. However, you might know that the “Gap” reports of UNEP are
usually dealing with the gaps in the efforts to counter climate change. From this
knowledge you may assume that Pairs 1 and 2 are correct. However in Pair 3, the question
of ‘production of what’ arises and should be dealt with carefully.

Pair 1 >> Correct


● The Emissions Gap Report is UNEP's spotlight report launched annually in advance of
the annual Climate negotiations. The EGR tracks the gap between where global
emissions are heading with current country commitments and where they ought to
be to limit warming to 1.5°C.

Pair 2 >> Correct


● The Adaptation Gap Report, published every year, examines how well the world is
preparing for climate change. It looks at the ways countries are planning, financing,
and implementing the projects they will need to endure droughts, floods, rising seas, and
other climate-related challenges. It also explores options for ramping up these climate
adaptation efforts.
● First published in 2014, the goal of the report is to inform policymakers and drive
national and international adaptation efforts.

Pair 3 >> Incorrect


● The Production Gap Report tracks the discrepancy between planned fossil fuel
production and global production levels consistent with limiting warming to 1.5°C
or 2°C.
● First published in 2019, the report is published by the Stockholm Environment Institute,
UNEP and other organizations.
Why this question?
The three climate-related ‘gap’ reports were released by UNEP in the run up
to the recent CoP 28.

30. C
Approach
● In this question, NBWL and NTCA are directly related to wildlife protection and hence
there are higher chances that both are correct. With the similar logic, NBA can be
eliminated because it is related to conservation of biodiversity; thus eliminating
Options A and D.

16
iPTS24 ENVIRONMENT ilearnoffc@gmail.com | 8089166792
Statement 1 >> Correct
● The NBWL is a statutory body that advises the government on wildlife conservation.
The Prime Minister chairs it. It has 30 members. It is constituted under the provisions
of Wildlife Protection Act.

Statement 2 >> Correct


● The National Tiger Conservation Authority (NTCA) is a statutory body set up by the
GoI in 2005 on the recommendations of the Tiger Task Force. It has been constituted
under the Wildlife Protection Act, 1972. A Director-General heads it, and it has 10
members. The Union Environment Minister chairs it.

Statement 3 >> Correct


● Community reserves were first introduced in the Wildlife (Protection) Amendment
Act of 2002 − the amendment to the Wildlife Protection Act of 1972. These categories
were added because of reduced protection in and around existing or proposed protected
areas due to private ownership of land, and land use.

Statement 4 >> Incorrect


● The National Biodiversity Authority (NBA) is a statutory autonomous body under the
Ministry of Environment, Forests and Climate Change, Government of India established
in 2003 to implement the provisions under the Biological Diversity Act, 2002, after
India signed Convention on Biological Diversity (CBD) in 1992.
Why this question?
Major statutes and its creations are frequently asked in UPSC Prelims.
Aspirants are advised to look into the origin of a body whenever you come
across one.

31. B
Approach
● Statement 1 and 2 are basic information regarding warm blooded and cold blooded
animals.
● In Statement 3, you need to be careful while reading the examples as there may be one
incorrect example among the correct examples.

Statement 1 >> Correct


● Warm-blooded is a term to describe animal species which have a relatively higher blood
temperature, and maintain thermal homeostasis primarily through internal metabolic
processes.

Statement 2 >> Correct


● Cold blooded animals do not use their metabolism to maintain body temperature.

Statement 3 >> Incorrect


● Snakes, lizards, skinks, turtles and tortoises are examples of cold blooded animals.
● Mammals and birds are examples of warm blooded animals.

17
iPTS24 ENVIRONMENT ilearnoffc@gmail.com | 8089166792
32. D
Approach
● This is a factual question. However you may find that Statement 1 is illogical because
UNESCO is more related to Education and Culture; eliminating Options A and C.

Statement 1 >> Incorrect


● UN-Habitat and UNEP, in collaboration with the Office of the President of the General
Assembly and the Republic of Türkiye, mark Zero Waste Day.
● The International Day of Zero Waste aims to promote sustainable consumption and
production patterns and raise awareness about how zero-waste initiatives contribute to
the advancement of the 2030 Agenda for Sustainable Development.

Statement 2 >> Incorrect


● The first-ever International Day of Zero Waste was celebrated on March 30, 2023, to
encourage people to prevent and minimise waste and promote a circular economy and
help achieve the Sustainable Development Goals.
Why this question?
The first-ever International Day of Zero Waste was celebrated on March
30, 2023.

33. D
Approach
● As India is rich in forest resources, it is highly unlikely that India imports such a large
quantity of wood; reaching Option D.

Statement 1 >> Incorrect


● India’s forests contribute just about five million cubic metres of wood every year. Almost
85 percent of the demand for wood and wood products is met by trees outside forests
(ToF). About 10 per cent is imported. India’s wood import bill is Rs 50,000-60,000
crore per year.

Statement 2 >> Correct


● India allows the export of only processed wood, not timber. In fact, the timber
harvested from Indian forests is not enough to meet the domestic demand for
housing, furniture, and other products. The demand for wood in India is 150-170 million
cubic metres annually, including 90-100 million cubic metres of raw wood. The rest goes
mainly towards meeting the demand for paper and pulp.
● Since ToF are so important, new certification standards are being developed for their
sustainable management.
Why this question?
The Ministry of Environment, Forests and Climate Change has recently
launched the Indian Forest & Wood Certification Scheme. This national
forest certification scheme offers voluntary third-party certification designed
to promote sustainable forest management and agroforestry in the country.
The scheme includes forest management certification, tree outside forest
management certification, and chain of custody certification.

18
iPTS24 ENVIRONMENT ilearnoffc@gmail.com | 8089166792
34. C
Approach
● This is a purely factual question. Attempt such questions with a clear idea about the facts.

Option C >> Correct


● Endorsed by more than 50 governments, the Restoration Barometer was developed
by IUCN with the support of the German Federal Ministry for the Environment, Nature
Conservation, Nuclear Safety and Consumer Protection.
● The Restoration Barometer (launched in 2016 as the Bonn Challenge Barometer) is the
only tool already used by governments to track the progress of restoration targets
across all terrestrial ecosystems including coastal and inland waters. It was designed
for countries that have committed to restore landscapes under international goals or
agreements.
Why this question?
Initiatives and outputs of various organisations are recurring themes of
UPSC Prelims. Aspirants are advised to go through their official websites to
collect useful information.

35. B
Approach
● Statements with figures are quite tricky to solve in prelims. Even then, it is important
to know and learn basic facts about important issues. With our basic knowledge about
emission scenarios, we can infer that both the statements are correct. Rest is basic
analysis whether statement 2 properly explains statement 1.

Statement 1 >> Correct


● The power sector is the largest emitter in India, accounting for 37 percent of total GHG
emissions, followed by the agricultural sector (21 percent), manufacturing (17
percent) and the transportation sector (9 percent)

Statement 2 >> Correct


● India became the world’s third-largest emitter of greenhouse gases (GHGs), after
China and the U.S. In 2021, it emitted 3.9 billion metric tons of carbon dioxide
equivalent (GtCO₂e), accounting for roughly seven percent of the global total.

36. D
Approach
● Relatively factual question. Carbon capture technologies are an important theme for
prelims.

Statement 1, 2, 3, 4 and 5 >> Correct


Cloud treatment to increase alkalinity
● Adding alkali to clouds or the ocean to enhance the reaction that sees CO, dissolve in
water, removing it from the air.

Enhanced ocean productivity


Adding iron or nitrogen to the ocean to increase the rate at which tiny microscopic plants

photosynthesise, thus accelerating their take up of atmospheric CO2.
19
iPTS24 ENVIRONMENT ilearnoffc@gmail.com | 8089166792
Blue carbon habitat restoration
● Conservation and restoration of degraded coastal and marine habitats, such as salt
marshes, mangroves, and seagrass beds, so they continue to draw CO, out of the air.

Building with biomass


● Using plant-based materials in construction, storing carbon and preserving it for as long
as the building remains standing

Afforestation and reforestation


● Planting trees where there were previously none (afforestation) or restoring areas where
the trees have been cut down or degraded (reforestation).

37. A
Approach
● Here, statement 1 can be solved easily as we do not count carbon monoxide in the primary
greenhouse gas list. Statement 2 is not an absolute or extreme statement and is worded
in such a way that it is possible to be true.

Statement 1 >> Correct


● Gases that trap heat in the earth’s atmosphere are known as Greenhouse Gases. The
Primary Greenhouse Gases are Water Vapour, Carbon dioxide, Methane, Nitrous oxide
and Ozone. Carbon monoxide (CO) is only a very weak direct greenhouse gas, but
has important indirect effects on global warming.

Statement 2 >> Correct


● CO is not considered a primary or significant greenhouse gas due to the weak (but
non-zero) absorption of energy in the infrared. However, it does increase global
warming by reacting with certain chemical species in the atmosphere which in turn lead
to an increase in concentration of primary greenhouse gases, most notably methane
and ozone.

38. C
Approach
● From common understanding we can establish that Bat, Dolphin and Whale use
echolocation for navigation.

Option 1 >> Correct


● Gangetic Dolphins can emit and receive the echoes of sound waves that bounce off any
objects near them in the water.

Option 2 >> Correct


● Bats produce echolocation by emitting high frequency sound pulses through their mouth
or nose and listening to the echo.

Option 3 >> Correct


● Whales have remarkable sensory ability for echolocation.

20
iPTS24 ENVIRONMENT ilearnoffc@gmail.com | 8089166792
Option 4 >> Incorrect
● Malabar Civet does not use Echolocation for navigation.
Additional information
Gangetic Dolphin
● Ganges River Dolphin prefer deep waters and in and around the confluence of rivers.
● They are indicator species of the health of freshwater.
● IUCN Status: Endangered
Why this question?
Uttar Pradesh declares Gangetic Dolphin as state aquatic animal.
Also based on PYQ

39. C
Approach
● This question requires knowledge of the topic because every statement contains a fact.

Statement 1 >> Correct


● This was a major finding that Invasive species play a key role in 60% of global plant and
animal extinctions and cost more than $400billion dollars to humanity per year.
● These Invasive species are one of the five major direct drivers of biodiversity loss The
other four are
 land and sea use change,
 direct exploitation of organisms,
 climate change
 Pollution.

Statement 2 >> Correct


● The water hyacinth is the world’s most widespread invasive alien species on land.
Lantana, a flowering shrub, and the black rat are the second and third most widespread
globally.

Statement 3 >> Correct


● It is headquartered in Bonn.

Additional information
● IPBES (Intergovernmental Science Policy Platform on Biodiversity and Ecosystem
Services.
 Intergovernmental Organisation
 HQ: Bonn, Germany
 Not a UN Body
Why this question?
The Intergovernmental Platform on Biodiversity and Ecosystem Services
(IPBES) has released its new publication – the “Assessment Report on
Invasive Alien Species and their Control’’.

21
iPTS24 ENVIRONMENT ilearnoffc@gmail.com | 8089166792
40. D
Approach
● This is a purely factual question. Attempt such questions with a clear idea about the facts.

Statement 1 >> Incorrect


● The International Coral Reef Initiative (ICRI) is an informal partnership among
nations, international organisations and non-government organisations to help
protect coral reefs globally.
● It was launched at the First Conference of the Parties of the Convention on Biological
Diversity, held in the Bahamas in December 1994, and subsequently announced at
the high level segment of the Intersessional Meeting of the United Nations Commission on
Sustainable Development in April 1995.

Statement 2 >> Incorrect


● ICRI was established in 1994 at the initiative of eight founding nations: Australia,
France, Japan, Jamaica, the Philippines, Sweden, the United Kingdom, and the
United States.
● The first General Meeting of members was held in Dumaguete, Philippines, in June 1995
where ICRI's foundational documents were adopted: its "Call to Action '' and "Framework
for Action '' for achieving sustainable management of coral reefs and related ecosystems.
● It is the "only global entity solely devoted to coral reefs".
Why this question?
The International Coral Reef Initiative (ICRI) and the Global Fund for Coral
Reefs (GFCR) recently launched the Coral Reef Breakthrough process
which aims to secure the future of at least 125,000 km2 of shallow-water
tropical coral reefs with investments of at least US$12 billion to support
the resilience of more than half a billion people globally by 2030.

41. C
Approach
● This is a factual question. However, the name of the authority itself suggests that it is a
powerful body. Hence, it is likely that it has the powers mentioned in Statement 2. With
the same logic you may take the risk of assuming Statement 1 as correct, because such
powerful bodies most often come with statutory backing.

Statement 1 >> Correct


● The Central Zoo Authority (CZA) is a statutory body in India created by the Wildlife
Protection Act of 1972. Its primary role is to regulate and oversee the functioning of
zoos in the country.
● The CZA sets standards and guidelines for the management of zoos, ensures the
welfare of animals, and works towards conservation and education initiatives related to
wildlife.
● Being a statutory body means it is established by law and has specific legal powers and
responsibilities defined by the legislation that created it.

Statement 2 >> Correct


● Every zoo in the country is required to obtain recognition from the Authority for its
operation. The Authority evaluates the zoos with reference to the parameters prescribed
under the Rules and grants recognition accordingly.
22
iPTS24 ENVIRONMENT ilearnoffc@gmail.com | 8089166792
Why this question?
CZA appears in news whenever there is a major event in any zoo, like
introducing lion safari parks, translocating animals from one to another zoo
etc.

42. C
Approach
● If you take a very broad view of the key term in the question, you can easily assume
that all the given statements will come under it; reaching Option C.

Statement 1, 2 and 3 >> Correct


● All the activities mentioned fall under the category of climate engineering, which is
also known as geoengineering. Climate engineering involves deliberate modifications of
the Earth's environment to mitigate the effects of climate change. In this case:
● Planting trees to restore forests is a form of climate engineering, specifically considered
as a part of "afforestation" or "reforestation" strategies to capture and store carbon
dioxide.
● Adding nutrients to the ocean to stimulate the growth of phytoplankton is another
climate engineering approach, often referred to as "ocean fertilisation." The idea is to
enhance carbon dioxide uptake by promoting the growth of phytoplankton, which can
absorb and store carbon.
● Injecting aerosols into the stratosphere to reflect a portion of incoming solar radiation
is a form of solar radiation management (SRM), a subcategory of climate engineering.
This method aims to cool the Earth by reflecting sunlight away from the planet.
● Therefore, all three activities fall under the umbrella of climate engineering.
Why this question?
Concepts related to conservation efforts are favourite topics of UPSC. A
thorough understanding of these concepts will help to tackle those
application level questions that appear in Prelims.

43. C
Approach
● This is a purely factual question. Attempt such questions with a clear idea about the facts.

Statement 1 and 2 >> Correct


● The Convention on Wetlands of International Importance especially as Waterfowl
Habitat’ (Convention on Wetlands) is an intergovernmental treaty that provides the
framework for national action and international cooperation for the conservation and wise
use of wetlands and their resources.
● It entered into force in 1975. It is neither legally binding and nor is a part of UN &
UNESCO conventions.
● The treaty was signed in 1971 at the Iranian town of Ramsar, on the southern shore of
the Caspian Sea. Since then, the Convention has been known as the Ramsar Convention.
● Currently, two wetlands of India are in Montreux record: Keoladeo Ghana National
Park, Rajasthan and Loktak Lake, Manipur.

23
iPTS24 ENVIRONMENT ilearnoffc@gmail.com | 8089166792
● Chilka Lake was placed on the Montreux Record in 1993 due to problems caused by
siltation and sedimentation. It was removed from the Record in 2002 following
rehabilitation efforts by the Chilika Development Authority.
Why this question?
Recently, the Ramsar Convention was in news related to its Wetland City
Accreditation (WCA) scheme.

44. A
Approach
● Most of the recent schemes that act on the ground level are trying to impact the demand
side of the issue through means like behavioural change campaigns, community
participation etc. Hence, Statement 2 is likely to be incorrect. Also, you might be aware of
severely water-stressed areas outside the northwest of India; hence, you may take a risk
of assuming Statement 3 as incorrect; eliminating Options B and C.

Statement 1 >> Correct


● Atal Bhujal Yojana (ATAL JAL) is a Central Sector Scheme for facilitating sustainable
groundwater management with an outlay of Rs. 6000 crore. Out of this, Rs. 3,000 is loan
from the World Bank and Rs. 3,000 crore is matching contribution from the Government
of India (GoI).

Statement 2 >> Incorrect


● The scheme lays emphasis on community participation and demand side
interventions for sustainable groundwater management in identified water stressed
areas in seven States of the Country.
● The scheme also envisages improved source sustainability for Jal Jeevan Mission, positive
contribution to the Government’s goal of ‘doubling farmer’s income’ and inculcating
behavioural changes in the community to facilitate optimal water use. Under the scheme,
focus is on awareness creation, capacity building, convergence with different
schemes and better agricultural practices.

Statement 3 >> Incorrect


● The scheme is being taken up in 8220 water stressed Gram Panchayats of 7 States,
namely, Haryana, Gujarat,Karnataka, Madhya Pradesh, Maharashtra, Rajasthan and
Uttar Pradesh.
Why this question?
Last year, there were questions directly from various government schemes
and there is high chance for that trend to continue.

45. D
Approach
● This is a purely factual question. Attempt such questions with a clear idea about the facts.

Statement 1, 2, 3 and 4 >> Correct


● The Global Environment Facility (GEF) is a financial mechanism that provides
funding to address global environmental issues. It serves as a partnership between
various countries, international institutions, NGOs, and the private sector. The GEF

24
iPTS24 ENVIRONMENT ilearnoffc@gmail.com | 8089166792
supports projects and programs related to biodiversity conservation, climate change
mitigation, international waters, land degradation, and chemicals and waste
management.

● Conventions under the GEF include:


i. Convention on Biological Diversity (CBD): Focused on the conservation and
sustainable use of biological diversity.
ii. United Nations Framework Convention on Climate Change (UNFCCC): Aims
to address climate change, including mitigation and adaptation efforts.
iii. United Nations Convention to Combat Desertification (UNCCD): Targets the
prevention and reversal of land degradation in arid, semi-arid, and dry sub-
humid areas.
iv. Stockholm Convention on Persistent Organic Pollutants (POPs): Aims to
eliminate or restrict the production and use of persistent organic pollutants.
v. Minamata Convention on Mercury: Seeks to protect human health and the
environment from the adverse effects of mercury.
● These conventions operate under the guidance of the GEF to facilitate international
cooperation and financial assistance for addressing global environmental challenges.
Why this question?
Last October was the 10th anniversary of the adoption of the Minamata
Convention.

46. C
Approach
● If you can correlate ‘tipping point’ with something that should not be ‘crossed’, the
most apt Option will be Option C because it discusses about a ‘threshold’.

Option C >> Correct


● In the context of the environment, a "tipping point" refers to a critical threshold where
a relatively small change in external conditions can lead to a significant and often
irreversible change in the state of a system.
● Crossing a tipping point can result in abrupt and potentially widespread transformations
in ecosystems or climate patterns. Once a tipping point is passed, the system may undergo
rapid and often unpredictable changes, making it difficult to return to its previous state.
Why this question?
Concepts related to the environment are favourite topics of UPSC. A
thorough understanding of these concepts will help to tackle those
application level questions that appear in Prelims.

47. C
Approach
● Statement 3 says, halophytic graminoids. Since we know that the Kutch region is a salty
marsh, this statement can be logically guessed as correct. Other statements are fact
based. However, it is a familiar topic as it appears in the news frequently.

Statement 1 >> Correct


● Banni Grassland consists of 2 ecosystems in juxtaposition of wetland and grasslands.

25
iPTS24 ENVIRONMENT ilearnoffc@gmail.com | 8089166792
Statement 2 >> Correct
● The region serves as breeding ground for Banni Buffaloes and Kankrej cows.

Statement 3 >>Correct
● It hosts halophytic graminoids.
● Graminoids are herbs with grass-like morphology.

Additional information
● It also suffers from the threat of Prosopis JuIiflora.(PYQ)
● The Banni grassland in Kutch district is a natural, unique and complex ecosystem. One
of the biggest grasslands in the Asian region spanning an area of 2,497 sq km
Why this question?
After the Kuno National park in Madhya Pradesh, the Gujarat government
plans to introduce a Cheetah conservation breeding centre in Kutch.

48. C
Approach
● Statement 1, logically followers. Ecologically Sensitive Zones may be declared by the
Ministry of Environment forest and climate change. Hydroelectric projects are a natural
Environmental threat. Hence may be prohibited. Rainwater harvesting and Organic
farming methods do not create pressure on Environment. Hence such activities may be
permitted.

Statement 1 >> Correct


● Eco-sensitive zones are ecologically Fragile Areas are areas within 10 km around
protected areas to act as a buffer. They are notified under section 3 of the Environment
Protection Act 1986 by the Ministry of Environment and Forest.
● Areas beyond 10 km can also be notified based on the sensitivity of the region.

Statement 2 >> Correct


● Commercial mining, saw mills, industries causing pollution (air, water, soil, noise etc),
establishment of major hydroelectric projects (HEP), commercial use of wood are
prohibited.

Statement 2 >> Correct


● Ongoing agricultural or horticultural practices, rainwater harvesting, organic farming, use
of renewable energy sources, adoption of green technology for all activities are permitted.

Additional information
● Regulated Activities: Felling of trees, establishment of hotels and resorts, commercial
use of natural water, erection of electrical cables, drastic change of agriculture system,
e.g., adoption of heavy technology, pesticides etc, widening of roads.
Why this question?
Based on the recent supreme Court Judgements

26
iPTS24 ENVIRONMENT ilearnoffc@gmail.com | 8089166792
49. C
Approach
● Statement 1 and statement 2 can be understood from the name itself. Statement 3 is also
a general statement. It uses the word possible, which enhances the possibility of species
survival. Hence can be taken as correct, which makes the answer Option C

Statement 1 >> Correct


● Genetic diversity is the number of different inherited traits of a species. It can be
observed within a species:

Statement 2 >> Correct


● Species diversity is defined as the number of different species present in an ecosystem
Species diversity consists of species richness, the number of species, and species
evenness, the relative abundance of species.

Statement 3 >> Correct


● Genetic Diversity is particularly important under rapid environmental change. Thus, large
genetic diversity – a big gene pool – positively affects ecosystem resilience and function.
When we drain species of their genetic diversity we destroy their adaptive potential, and
their long-term survival will be jeopardised.
Additional information
● Diagrams below can be used for better clarity.

27
iPTS24 ENVIRONMENT ilearnoffc@gmail.com | 8089166792
50. A
Approach
● This question is fact based. But the facts are based on NCERT. Hence it is important to
know.

Statement 1 >> Incorrect


● Only 1500 species of Vascular plants are required to be endemic to be treated as
Biodiversity hotspot.

Statement 2 >> Correct


● It must have lost at least 70% of original habitat.

Statement 3 >>Incorrect
● There are 4 identified hotspots in India
1. Himalayas
2. Indo Burma
3. Sundaland
4. Western Ghats and Sri Lanka

Additional information
● Biodiversity Hotspots was defined by Norman Myers. His definition was adopted by
Conservation International.
● Accordingly the following 2 criteria must be met by hotspots
1. It must contain at least 1,500 species of vascular plants (> 0.5% of the world’s total)
as endemic.
2. It must have lost at least 70% of its original habitat. (It must have 30% or less of
its original natural vegetation.

51. C
Approach
● If we know that Guindy national park is located in Tamil Nadu there we can assume that
it has a Tropical dry evergreen forest (using the knowledge from Geography). Also we know

28
iPTS24 ENVIRONMENT ilearnoffc@gmail.com | 8089166792
that Tamil Nadu is a state which receives rainfall from the South West and North east
Monsoon. Hence we can take the second statement also as correct.Since we learn species
distribution we know that Black buck is an antelope that is distributed throughout India.
Tortoise can be guessed as Tamil Nadu is a coastal state.

Statement 1 >> Correct


● It is tropical dry evergreen forest

Statement 2 >>Correct
● National park receives rainfall in summer and winter.
● The dissymmetric climatic condition of this region, where rainfall occurs both during
summer and winter monsoons and extended the dry season from March to September,
favours the development and sustenance of the Tropical Dry Evergreen forest in this
narrow strip.

Statement 3 >> Correct


● It is home to Black bucks, Tortoises etc

Additional information
● The national park is located in the heart of Chennai metropolitan Area.
Why this question?
Oil spill in Guindy National park

52. C
Approach
● Cruelty is done, where animals are used on a large scale. Considering that case, the
Ministry of Fisheries, animal Husbandry and dairying deals with a wide range of activities
including slaughter. Hence Option C can be the answer

Animal Welfare Board of India


● The Animal Welfare Board of India is a statutory advisory body on Animal Welfare Laws
and promotes animal welfare in the country.
● Established in 1962 under Section 4 of the Prevention of Cruelty to Animals Act, 1960,
the Animal Welfare Board of India was started under the stewardship of Late Smt.
Rukmini Devi Arundale, well known humanitarian.
● Board ensures that animal welfare laws in the country are diligently followed, provides
grants to Animal Welfare Organizations and advises the Government of India on animal
welfare issues.
● The Board consists of 28 Members including 6 Members of Parliament (2 Members of
Parliament from Rajya Sabha and 4 Members of Parliament from Lok Sabha).
● The term of office of Members is for a period of 3 years

Additional information
● Organisations under Ministry of Environment forest and Climate Change
 Central Zoo authority, Botanical survey of India,Zoological survey of India
 National Board of Wildlife
 National Tiger Conservation Authority
 Wildlife Crime control Bureau
29
iPTS24 ENVIRONMENT ilearnoffc@gmail.com | 8089166792
 National Biodiversity Authority
 Genetic Engineering Appraisal Committee.
Why this question?
Questions based on Ministries that Implement a particular Act was asked
Previously.(2021,2018,2014,2007)

53. B
Approach
● It's a direct factual question. Climate related reports are sometimes asked in the prelims.

Statement 2 >> Correct


● WMO publishes the Annual Greenhouse Gas Bulletins. Each year, these bulletins will
report the latest trends and atmospheric burdens of the most influential, long-lived
greenhouse gases; carbon dioxide (CO2), methane (CH4), and nitrous oxide (N2O),
as well as a summary of the contributions of the lesser gases.

54. D
Approach
● This question is fact based. However if we have visited the tables of ISFR report, these
major data will be seen.

Statement 1 >> Incorrect


● The report is released by the Forest Survey of India.
Statement 2>> Incorrect
● Arunachal Pradesh recorded a net decrease. (-257%)

Statement 3 >> Incorrect


● Lakshadweep has the largest percentage of area under forest cover which is above 90%.
● In case of Goa it is 60%

Additional information
● FSI
 Under Ministry of Environment Forest and Climate Change
 Established in 1981, Dehradun.
 ISFR is a Biennial Report released since 1987

Why this question?


ISFR was last published in 2021. New report is likely to be published and it
is one of the most important reports.

55. B
Approach
● This question is fact based. But based on NCERT.

30
iPTS24 ENVIRONMENT ilearnoffc@gmail.com | 8089166792
Statement 1 >> Incorrect
● More than 70 per cent of all the species recorded are animals, while plants (including
algae, fungi, bryophytes, gymnosperms and angiosperms) comprise no more than 22
percent of the total.

Statement 2 >> Correct


● Among animals, insects are the most species-rich taxonomic group, making up more than
70 per cent of the total. That means, out of every 10 animals on this planet, 7 are insects.

Statement 3 >> Correct


● The number of fungi species in the world is more than the combined total of the species
of fishes, amphibians, reptiles and mammals.

56. C
Approach
● War is not a global phenomenon, in order to propel a huge biodiversity loss. Hence
Statement 6 can be eliminated. Then we reach a 50:50.

Evil Quartet
1. Habitat loss and fragmentation: This is the most important cause driving animals
and plants to extinction. The most dramatic examples of habitat loss come from
tropical rainforests.
2. Over-exploitation:: Many species were extinct from earth because of over
exploitation.Eg: Stellar Sea Cow, Passenger Pigeon
3. Alien species invasions: When alien species are introduced unintentionally or
deliberately for whatever purpose, some of them turn invasive, and cause decline
or extinction of indigenous species. The Nile perch introduced into Lake Victoria in
east Africa led eventually to the extinction of an ecologically unique assemblage of
more than 200 species of cichlid fish in the lake.
4. Co-extinctions: When a species becomes extinct, the plant and animal species
associated with it in an obligatory way also become extinct. When a host fish
species becomes extinct, its unique assemblage of parasites also meets the same
fate.

57. C
Approach
● Core Zone is an extremely protected area. But there are chances that such an area is
included under scientific research.

Statement 1 >> Correct


● Biosphere Reserves are designated under the intergovernmental MAB Programme by the
Director-General of UNESCO following the decisions of the MAB International
Coordinating Council (MAB-ICC).

Statement 2 >> Correct


●3 Core functions:
● Conservation of biodiversity and cultural diversity
● Economic development that is socio-culturally and environmentally sustainable
● Logistic support underpinning development through research, monitoring,
education and training.
31
iPTS24 ENVIRONMENT ilearnoffc@gmail.com | 8089166792
Statement 3 >> Correct
● These are legally protected sites for conserving biological diversity, monitoring minimally
disturbed ecosystems, undertaking non-destructive research and other low-impact uses
(e.g. education), and so on. Core areas are generally natural or near natural areas, or
areas with a high level of biodiversity. They provide an example of what a specific
ecosystem would look like in the absence of – or with only minimal – human interference,
or the result of a long-term specific human-nature relationship.

Additional information
● Biosphere reserves are nominated by national governments and remain under the
sovereign jurisdiction of the states where they are located. Their status is internationally
recognized. Member States can submit sites through the designation process.
● These three functions are pursued through the Biosphere Reserves' three main zones.
○ Core areas
They comprise a strictly protected zone that contributes to the conservation of
landscapes, ecosystems, species and genetic variation
○ Buffer zones
They surround or adjoin the core area(s), and are used for activities compatible
with sound ecological practices that can reinforce scientific research, monitoring,
training and education.
○ Transition area
The transition area is where communities foster socio-culturally and ecologically
sustainable economic and human activities.

Note: There are 18 Biosphere reserves designated by Government of India of this


Only 12 are MAB sites
Why this question?
On November 3, the world celebrates the International Day for Biosphere
Reserves, a day established by UNESCO during its 41st General Conference
in 2022.

58. C
Approach
● This question is fact based.

32
iPTS24 ENVIRONMENT ilearnoffc@gmail.com | 8089166792
Statement 1 >>Correct
● Critical Tiger Habitat is defined under Wildlife Protection Act, 1972.
● Based on deliberations with experts and simulation results from scientific data, it has
been found that a minimum inviolate area of 800-1200 sq km is required to sustain a
viable population of tigers (20 breeding females).

Statement 2 >> Correct


● Declared under Forest Rights act.

Statement 3 >> Correct


● Informed consent of Gram Sabha is required

Additional information

Core or Critical Tiger Habitat Critical WildLife Habitat

Mentioned in Wildlife protection Mentioned in Forest Rights Act,2006


Act,1972

Notified by state Government with the Declared by Central


consultation of an Expert Committee Government(Ministry of Environment
Forest and Climate Change)

Why this question?


NTCA declares that Nugu WildLife Sanctuary of Bandipur Tiger Reserve to
be declared as a core critical tiger Habitat.

59. A
Approach
● Consider the statement 2. We may think that for accessing a medicinal plant in our
locality we will not gain permission from the Biodiversity Management Committee. Hence
we can arrive at the conclusion that, statement 2 may be Incorrect.

Statement 1 >> Correct


● As per Section 37 (1) of the Biological Diversity Act,2002 the State Government may, from
time to time in consultation with the local bodies, notify in the Official Gazette, of areas
of biodiversity importance as Biodiversity Heritage Sites.

Statement 2 >> Incorrect


● There is no such provision. Local people and communities of the area, including Vaids
and Hakims will have free access to use biological resources within the country for their
own use, medicinal purposes and research.

33
iPTS24 ENVIRONMENT ilearnoffc@gmail.com | 8089166792
Additional information
Biological Diversity Act, 2002
● This Act provides for setting up of a National Biodiversity Authority (NBA), State
Biodiversity Boards (SBB) and Biodiversity Management Committees (BMC) in local
bodies.
● All foreign nationals’ organisations require prior approval of NBA for obtaining biological
resources and/or associated knowledge for any use.
● Similarly, Indian nationals or organisations will be required to give prior intimation to the
concerned SBB about any biological resources being imported for commercial use. The
SBB may prohibit the import if found to violate the objectives of conservation, sustainable
use and benefit sharing.
● However, local people and communities of the area, including Vaids and Hakims will
have free access to use biological resources within the country for their own use,
medicinal purposes and research.
● While granting approvals, NBA will impose terms and conditions to secure equitable
sharing of benefits.
● There is an enabling provision for setting up a framework for protecting traditional
knowledge.
● The monetary benefits, fees and royalties, as a result of approvals by NBA are to be
deposited in the National Biodiversity Fund which will be used for conservation and
development of areas from where the resource has been accessed, in consultation with
local self-government.
● Under Section 37 of Biological Diversity Act, 2002 the State Government in
consultation with local bodies may notify the areas of biodiversity importance as
Biodiversity Heritage Sites.
● The Biodiversity Heritage Sites are the well-defined areas that are unique, ecologically
fragile ecosystems - terrestrial, coastal and inland waters and, marine having rich
biodiversity.
60. C
Approach
● Here statement 1 can be solved easily. Statement 2 is a data based question. In exams,
see whether such datas are extreme or not. 70% is too much carbon dioxide that can
destroy the ocean ecosystem.

Statement 1 >> Correct


● Ocean acidification is the process in which seawater becomes more acidic because of
the excess carbon dioxide (CO2) it is absorbing from the atmosphere.
● When carbon dioxide dissolves into the ocean, CO2 and water produce carbonic acid,
which releases hydrogen and bicarbonate ions. The more hydrogen ions there are,
the more acidic the water becomes. This is what drives ocean acidification.

Statement 2 >> Correct


● The ocean holds 60 times more carbon than the atmosphere and absorbs almost 30%
of carbon dioxide (CO₂) emissions from human activities.

61. C
Approach
● Aspirants are advised to have knowledge about such facts from NCERTs.

34
iPTS24 ENVIRONMENT ilearnoffc@gmail.com | 8089166792
Statement 1 >> Correct
● A Detritus food chain is based on autotrophs energy capture initiated by grazing animals.
Statement 2 >> Correct
● It involves the decomposition of organic wastes and dead matter received from grazing
food chains.

Additional information

Why this question?


Statement is directly picked from NCERT Class 11, Geography, Chapter
15

62. B
Approach
● This question is fact based. However, since land is an under State list, it can be guessed
that wetland declaration is under the purview of the state government. However this is
not a reliable guess.

Statement 1 >> Incorrect


● Wetlands declared under Wetland Management rules cannot Include man made water
bodies. However Ramsar wetlands can include man made water bodies.

Statement 2 >>Correct
● The following cannot be declared as wetlands under Wetland Rules,2017
 River channels, paddy fields, human-made water bodies, aquaculture, salt
production, recreation, irrigation projects,
 wetlands and protected areas falling within areas covered under the Indian Forest
Act, 1927, Forest (Conservation) Act, 1980, Wildlife (Protection) Act, 1972 and
 The Coastal Regulation Zone Notification, 2019 cannot be notified under the
Wetlands Rules 2017.

Additional information
● Wetland Management Rules 2017
 These rules decentralised wetlands management by giving states/UTs powers to
not only identify and notify wetlands within their jurisdictions but also keep a
watch on prohibited activities.
 The rules provide an advisory role for the National Wetland Committee to guide
the state bodies on the integrated management of wetlands based on the wise‐use
principle and review the progress of integrated management of Ramsar Convention
sites, among other roles.
 To oversee the work carried out by States, the rules stipulate setting up the
National Wetlands Committee (NWC – headed by the MoEF Secretary). NWC is the
35
iPTS24 ENVIRONMENT ilearnoffc@gmail.com | 8089166792
nodal advisory body for the National Plan for Conservation of Aquatic Eco-systems
(NPCA). It guides the state bodies on the integrated management of wetlands based
on the wise‐use principle.

63. D
Approach
● With Basic understanding of Biodiversity protocols we can arrive at the answer. The term
Living Modified Organisms and Biosafety clearing house is related to Cartagena protocol
on Biosafety.

Statement 1 >> Incorrect


● The Cartagena Protocol on Biosafety to the Convention on Biological Diversity is an
international agreement which aims to ensure the safe handling, transport and use of
living modified organisms (LMOs) resulting from modern biotechnology that may have
adverse effects on biological diversity, taking also into account risks to human health. It
was adopted on 29 January 2000 and entered into force on 11 September 2003.

Statement 2 >> Incorrect


● The Cartagena Protocol also sets up a Biosafety Clearing-House (BCH) to enable
information exchange on LMOs between countries. The BCH is an information-sharing
mechanism for relevant technical, scientific and legal information.

Additional information
● Nagoya Protocol
 The Nagoya Protocol on Access to Genetic Resources and the Fair and Equitable
Sharing of Benefits Arising from their Utilisation to the Convention on Biological
Diversity is an international agreement which aims at sharing the benefits arising
from the utilisation of genetic resources in a fair and equitable way. It entered into
force on 12 October 2014.
 Access and Benefit Sharing (ABS) is the keyword of convention.

64. B
Approach
● Chances of green hydrogen being produced from fossil fuel is less. Hence can be
eliminated

Pair 1 >> Incorrect


● Grey hydrogen is created from natural gas, or methane, using steam methane reformation
but without capturing the greenhouse gases made in the process. Grey hydrogen is
essentially the same as blue hydrogen, but without the use of carbon capture and storage.

Pair 2 >> Incorrect


● Green hydrogen is made by using clean electricity from surplus renewable energy sources,
such as solar or wind power, to electrolyse water.

Pair 3 >> Correct


● Yellow hydrogen is a relatively new phrase for hydrogen made through electrolysis using
solar power Forest and Climate Change.

36
iPTS24 ENVIRONMENT ilearnoffc@gmail.com | 8089166792
Pair 4 >> Correct
● Turquoise hydrogen is made using a process called methane pyrolysis to produce
hydrogen and solid carbon. In the future, turquoise hydrogen may be valued as a low-
emission hydrogen, dependent on the thermal process being powered with renewable
energy and the carbon being permanently stored or used.

Additional information
White hydrogen
● White hydrogen is a naturally occurring, geological hydrogen found in underground
deposits and created through fracking. There are no strategies to exploit this hydrogen
at present.
Black and brown hydrogen
● Using black coal or lignite (brown coal) in the hydrogen-making process, these black and
brown hydrogen are the absolute opposite of green hydrogen in the hydrogen spectrum
and the most environmentally damaging.
● Just to confuse things, any hydrogen made from fossil fuels through the process of
‘gasification’ is sometimes called black or brown hydrogen interchangeably.
Why this question?
India’s first National Hydrogen fuel cell Bus launched

65. B
Approach
● Statement 2 is a general statement and can be assumed to be correct. Statement 1 is fact
based. But the assumption that can be taken here is, there are other bodies in the world
that work exclusively on desertification. Hence UNEP is less probable. But it is not an
accurate guess. Hence we can arrive at the answer as Option B.

Statement 1 >> Incorrect


● It is an initiative of the African Union.
● Launched in 2007
● Aim to transform degraded landscapes in the Sahara and Sahel region.

Statement 2 >> Correct


● Objectives include
○ fertile land
○ economic opportunities for the world’s youngest population
○ food security for the millions that go hungry every day
○ climate resilience in a region where temperatures are rising faster than anywhere
else on Earth
○ a new world wonder spanning 8000km (Reclamation of degraded land)

Additional information
● The Great Green wall Initiative
○ It aims to restore 100 million hectares of currently degraded land. Also, the project
envisages sequestering 250 million tons of carbon and creating 10 million green
jobs by 2030.
● There is a Green Wall project in India for the protection of Aravalli.

37
iPTS24 ENVIRONMENT ilearnoffc@gmail.com | 8089166792
66. D
Approach
● It is a difficult question which involves scientific explanation for ozone hole formation.
Here, one who has learned the reasons for Antarctic hole formation, probably can tweak
that knowledge to guess the answer.

Option A >> Incorrect


● The range of winter minimum temperatures found in the Arctic is much greater
than that in the Antarctic. Hence for some years, PSC (Polar Stratospheric Cloud)
formation temperatures are not reached in the Arctic, and significant ozone depletion
does not occur.

Option B >> Incorrect


● Winter temperatures are low enough for PSCs (Polar Stratospheric Cloud) to form
somewhere in the Antarctic for nearly the entire winter (about 5 months), and only for
limited periods (10–60 days) in the Arctic for most winters.
● Reactions on PSCs cause the formation of the highly reactive gas chlorine
monoxide (ClO), which is very effective in the chemical destruction of ozone.

Option C >> Incorrect


● The Southern Hemisphere polar vortex circulation tends to be stronger than that
in the Northern Hemisphere because northern polar latitudes have more land and
mountainous regions than southern polar latitudes. This situation leads to more
meteorological disturbances in the Northern Hemisphere, which increase the mixing of
air from lower latitudes that warms the Arctic stratosphere.

Option D >> Correct


● Significant denitrification occurs each winter in the Antarctic and only for
occasional winters in the Arctic, because PSC formation temperatures must be
sustained over an extensive altitude region and time period to lead to denitrification.
Denitrification removes the NOx available for converting the highly reactive chlorine gas
ClO back into the reservoir gas ClONO2. As a result, ClO remains chemically active
for a longer period, thereby increasing chemical ozone destruction.

67. B
Approach
● Location of Tiger Reserves are very important. This question can be answered only with
the information on Tiger Reserves and Rivers of India.
Pair 1 >> Correct
● Veerangana Durgawati Tiger reserve is in Narmada Basin.
● It is the 7th Tiger reserve in Madhya Pradesh.
● Spread across 3 districts.(Sagar, Damoh and Narsinghpur districts)
● Parts of the Reserve fall under Narmada and Yamuna Basin.

Pair 2 >> Correct


● Ramgarh Vishdhari is in Chambal Basin.
● It lies in the south-eastern part of Rajasthan in Bundi district with representation of both
Vindhyan and Aaravalli elements.

38
iPTS24 ENVIRONMENT ilearnoffc@gmail.com | 8089166792
● It is in continuation with the buffer area of Ranthambore tiger reserve in the North-eastern
side and the Mukundara Hills tiger reserve in the southern side.
● The Mez River (a tributary of Chambal River) passes through this tiger reserve.

Pair 3 >> Incorrect


● Dholpur Karauli Tiger Reserve is in Rajasthan.
● 5th in Rajasthan after Ranthambore, Sariska, Mukundra Hills and Ramgarh Vishdhari.

Additional information
● Tiger Reserves are notified by State Governments as per provisions of Section 38V of the
Wildlife (Protection) Act, 1972 on advice of the National Tiger Conservation Authority.

68. D
Approach
● This question can be tackled easily if we know the concept of Carbon Sequestration. In
ocean carbon sequestration, we need methods that can capture carbon/store
carbon/convert atmospheric carbon to other forms.

Statement 1 >> Correct


● In Artificial downwelling, surface water is transferred to the deep ocean along with
carbon. It could be achieved through pumps, artificially cooling surface waters, or
increasing salinity through thickening of ice.

Statement 2 >> Correct


● Captured CO2 could be stored in the ocean water column or on the sea bed. Captured
CO2 could also be mineralized via reaction with seafloor rocks, enabling permanent
carbon storage.

Statement 3 >> Correct


● Ocean alkalinity enhancement refers to carbon removal by adding alkaline substance to
sea water to accelerate the ocean’s natural carbon sink.

Statement 4 >> Correct


● Fertilising surface waters with nutrients like iron, phosphorus and nitrogen can
stimulate photosynthesis and cause phytoplankton to grow, which absorb CO2.

Statement 5 >> Correct


● Microalgae converts dissolved CO2 into organic carbon through photosynthesis.

69. D
Approach
● To answer this question, it is important to apply the concept of climate resilient
agriculture to real life. Climate resilient agriculture incorporates good agricultural
practices which incorporate efficient resource utilisation and adapting to changing
climatic conditions. So such methods need to be picked from the statements.

Statement 1 >> Correct


● One of the most critical and proactive measures that can be taken to cope with food
insecurity caused by unpredictable weather patterns is for farmers to adopt climate-

39
iPTS24 ENVIRONMENT ilearnoffc@gmail.com | 8089166792
resilient crops. Climate-resilient crops and crop varieties have enhanced tolerance to
biotic and abiotic stresses.

Statement 2 >> Correct


● A laser land leveller- a machine equipped with a laser guided drag bucket- is much
more effective and quicker in ensuring a flat table-top like surface. An even land means
irrigation water reaches every part of the field with minimal waste from run-off or
water-logging.

Statement 3 >> Correct


● Deficit irrigation is an optimization strategy in which irrigation is applied during
drought-sensitive growth stages of a crop. Outside these periods, irrigation is
limited or even unnecessary if rainfall provides a minimum supply of water. Total
irrigation application is therefore not proportional to irrigation requirements throughout
the crop cycle.

70. A
Approach
● The G7 climate club was in the news owing to COP 28. Here, Statement 1 is a general
one and can be tackled easily. During and after the COP 28 summit, there were
discussions demanding India to join the club. Hence statement 2 can be eliminated.

Statement 1 >> Correct


● The G7 Climate Club is a global climate conservation initiative launched by German
Chancellor Olaf Schoolz in December 2022 and inaugurated at COP28 2023. It aims
to help developing nations cut down on their industry emissions and curb the spread of
the climate crisis thereby speeding up the transition towards net-zero emissions by 2050
and achieve the goals drafted during the 2015 Paris Agreement.

Statement 2 >> Incorrect


● While countries like Australia, Mozambique, and Germany have made alliances to be
part of the Climate Club, the largest producers in the Steel industries, China and
India have decided to not gain access to the Climate Club Alliance.

71. B
Approach
● Statement 2 and 3 can be tackled easily with the basic knowledge of Adaptation Fund.
For Statement 3, we will have to take chance.

Statement 1 >> Correct


● The Adaptation Fund (AF) was established in 2001 to finance concrete adaptation
projects and programmes in developing countries that are parties to the Kyoto Protocol
and are particularly vulnerable to the adverse effects of climate change

Statement 2 >> Correct


● The Fund is financed in part by government and private donors, and also from a two
percent share of proceeds of Certified Emission Reductions (CERs) issued under the
Protocol’s Clean Development Mechanism projects.

40
iPTS24 ENVIRONMENT ilearnoffc@gmail.com | 8089166792
Statement 3 >> Incorrect
● The Green Climate Fund (GCF) – a critical element of the historic Paris Agreement - is
the world’s largest climate fund.

72. C
Approach
● Here, statement 1 is easy to solve. Analyse statement 2 by keeping in mind that average
day temperature in India during summer is around 35 degree Celsius.

Statement 1 >> Correct


● A Heat Wave is a period of abnormally high temperatures, more than the normal
maximum temperature that occurs during the summer season in the North-Western
parts of India. Heat Waves typically occur between March and June, and in some rare
cases even extend till July.

Statement 2 >> Correct


● The Indian Meteorological Department (IMD) has given the following criteria for Heat
Waves :
● Heat Wave need not be considered till maximum temperature of a station reaches at
least 40°C for Plains and at least 30°C for Hilly regions

73. B
Approach
● The Government has announced the National Green Hydrogen mission recently. So, one
who followed the developments can comfortably tackle statement 2 easily. Moreover, if
India were the first nation to define the green hydrogen standard, it would have been a
much widely publicised news. But we haven’t come across such information anywhere.
So can take a chance and eliminate Statement 1

Statement 1 >> Incorrect


● For the progress of the National Green Hydrogen Mission, the Ministry of New and
Renewable Energy (MNRE) has notified the Green Hydrogen Standard for India. It
outlines the emission thresholds that must be met in order for hydrogen produced to be
classified as ‘Green’, i.e., from renewable sources. With this notification, India
becomes one of the first few countries in the world to announce a definition of
Green Hydrogen.

Statement 2 >> Correct


● The new standard states that to be classified as green, hydrogen produced in India must
have well-to-gate emissions of no more than 2kg CO₂ equivalent per kilogram of
hydrogen produced. Well-to-gate refers to the entire production cycle from feedstock
through to production itself, which means that water treatment, electrolysis, gas
purification, drying and compression of hydrogen are all included in this.

74. A
Approach
● The logic involves understanding the provisions of the E-Waste (Management) Rules,
2022, and recognizing which statements align with those provisions. If you know

41
iPTS24 ENVIRONMENT ilearnoffc@gmail.com | 8089166792
radioactive wastes are not covered under the rules, you will be able to eliminate options
with statement 3.

Statement 1 >> Correct


● The Central Pollution Control Board (CPCB) will conduct random sampling of electrical
and electronic equipment.

Statement 2 >> Correct


● The rules mandate the reduction of hazardous substances, including lead, in the
manufacturing of electronic equipment.
● Manufacturers of electronic equipment are mandated to reduce the use of lead, mercury,
cadmium and other others that can harm human health and environment.

Statement 3 >> Incorrect


● The E-Waste (Management) Rules, 2022 do not apply to radio-active wastes.
● Radio-active wastes are covered under the Atomic Energy Act, 1962.

Additional information
● The E-Waste (Management) Rules, 2022 was published by the Ministry of Environment,
forest and climate change on November 2, 2022.
● They will apply to all businesses and individuals involved in manufacturing, sales,
transfer, purchase, refurbishing, dismantling, recycling and processing of e-waste or
electrical and electronic equipment.
● Under the new rules, the number of items that have been categorized as e-waste has been
increased from 21 to 106.
● It includes all electrical devices and radiotherapy equipment, nuclear medicine equipment
and accessories, Magnetic Resonance Imaging (MRI), electric toys, air conditioners,
microwaves, tablets, washing machine, refrigerator, iPad and others.
● This includes electronic components, consumables, parts and spares that make the
electronic products operational.
● The new rules are not applicable for waste batteries, which are covered under the
Battery Waste Management Rules, 2022.
● It is also not applicable for packaging plastics, which are covered under the Plastic Waste
Management Rules, 2016.
● It also does not apply for micro enterprises and radio-active wastes, which are covered
under the Micro, Small and Medium Enterprises Development Act, 2006 and Atomic
Energy Act, 1962 respectively.
● The rules restrict the use of hazardous substances for manufacturing electrical and
electronic equipment. This comes in response to the deaths caused by exposure to
radioactive materials.
● Manufacturers are required to use technologies and methods that make the end product
recyclable. They are also required to ensure the compatibility of components or parts
developed by different manufacturers. This will minimise the generation of e-wastes.
● Imports or sales of new electrical and electronic equipment are allowed only if they comply
with the government regulations. If the product does not comply with the rules, the
manufacturer must withdraw all samples from the market.
Why this question?
Solid waste management rules was asked in UPSC prelims 2019.

42
iPTS24 ENVIRONMENT ilearnoffc@gmail.com | 8089166792
75. C
Approach
● In this question, the aspirants are only expected to identify that statement 2 is incorrect.
Hence, it was a topic that often appeared in the news. It would be an easy task.

Statement 1 >> Correct


● The method of injecting pressurised air or gas into a contaminated area to increase in-
situ aerobic biological activity is known as biosparging.

Statement 2 >> Incorrect


● In bioventing, only carbon dioxide and clean air are vented and no volatile organic
hydrocarbons.
● On the other hand, biosparging involves applying air under pressure at low rates of
flow to groundwater. This is to increase oxygen levels in groundwater and enhance
aerobic biodegradation of organic contaminants. Mixing is increased in saturated areas,
meanwhile increasing the contact between groundwater and soil.

Additional information
● According to saturation and aeration levels of a contaminated area (soil or water), different
in situ and ex situ bioremediation strategies can be applied. In situ techniques include
bioventing, biosparging, biostimulation, and bioaugmentation, whereas ex situ ones
involve biopiles, bioreactors, biofilters, landfarming, and composting.
● Biosparging is an in-situ remediation method that employs indigenous microorganisms
to break down organic components in the saturated zone.
● Biosparging involves injecting air (or oxygen) and nutrients (if needed) into the saturated
zone to boost the biological activity of the local microorganisms.
● Petroleum compounds dissolved in groundwater, adsorbed to soil below the water table,
and inside the capillary fringe can all be reduced by biosparging.
● The process of biosparging is identical to that of air sparging.
Why this question?
As a part of a recent study, a full-scale biosparging investigation was
conducted at a petroleum-hydrocarbon spill site.

76. B
Approach
● The logic involves recognizing the specific effects and characteristics associated with each
term and linking them to the appropriate type of pollution. Understanding these
components and their relation to the impact of artificial light on the environment helps
identify light pollution as the correct category.

Option B >> Correct


Components of light pollution include:
● Glare: Excessive brightness that causes visual discomfort
● Skyglow: Brightening of the night sky over inhabited areas
● Light trespass: Light falling where it is not intended or needed
● Clutter: Bright, confusing and excessive groupings of light sources.

43
iPTS24 ENVIRONMENT ilearnoffc@gmail.com | 8089166792
Additional information
● The inappropriate or excessive use of artificial light – known as Light Pollution (LP)
– can have serious environmental consequences for humans, wildlife, and our climate.
● Light Pollution is a side effect of industrial civilization.
● Its sources include building exterior and interior lighting, advertising, commercial
properties, offices, factories, streetlights, and illuminated sporting venues.
● The Skyglow, is an omnipresent sheet of light across the night sky in and around cities
that can block all but the very brightest stars from view.
● The brightening of the night sky over inhabited areas because of streetlights, security
floodlights and outdoor ornamental lights cause the Skyglow.
● This light floods directly into the eyes of the Nocturnal (active at night) and also into
the skies and misleads their path.
● 'Skyglow' is one of the components of light pollution.

Why this question?


Recently, a new study has found that non-natural light had increased the
brightness of Skyglow, by 9.2-10% every year between 2011 and 2022 with
significant ecological, health and cultural implications.

77. A
Approach
● More than knowledge, the question tests the commonsensical approach of a candidate.
Acid rain causing skin burn is unlikely and can be eliminated.

Statement 1 >> Correct


● Acid rain does not directly affect human health. The acid in the rainwater is too
dilute to have direct adverse effects. The particulates responsible for acid rain
(sulphur dioxide and nitrogen oxides) do have an adverse effect. These particulates come
together and react in the atmosphere, forming nitrate particles and fine sulphate.
Exposure to high amounts of these particles, particularly over a long period of
time, can cause irritation to the eyes and skin.

Statement 2 >> Correct


● The pollution that causes acid rain can also create tiny particles. When these particles
get into people’s lungs, they can cause health problems, or can make existing health
problems worse. Also, nitrogen oxides cause ground-level ozone. This ground-level
ozone causes respiratory problems, like pneumonia and bronchitis, and can even cause
permanent lung damage. The health effects are caused when people breathe in these
tiny particles or ozone.

Statement 3 >> Incorrect


● Acid rain is never acidic enough to burn human skin. However, very strong acids
can, and will, burn the skin on contact, and can even destroy some metals. But for this
to occur, the pH of these kinds of acids needs to be very low, around pH 1.

Statement 4 >> Correct


● Acid rain falls at a pH value of roughly 4, making it harmful to humans. With acidic pH
levels in humans comes hair loss, low urinary pH, severe mineral imbalances and
constipation.

44
iPTS24 ENVIRONMENT ilearnoffc@gmail.com | 8089166792
Statement 5 >> Correct
● Exposure to high amounts of particles like nitrate particles and fine sulphate
particularly over a long period of time, can cause irritation to the eyes and skin

78. B
Approach
● The majority of plastics, including PVC (Polyvinyl Chloride), are produced from synthetic
monomers derived from petrochemical sources. In the case of PVC, the main raw material
is vinyl chloride, which is typically synthesized from ethylene, a hydrocarbon obtained
from natural gas or petroleum. Hence vinyl chloride can't occur naturally. Hence
statement 1 is incorrect. Amorphous materials often have a higher affinity for
incorporating or reacting with other substances due to lack of a regular pattern. Hence
statement 4 is incorrect.

Statement 1 >> Incorrect


● Vinyl chloride is a colourless gas that burns easily.
● It is produced industrially for its commercial uses.
● It can’t occur naturally. Used primarily to make polyvinyl chloride (PVC).

Statement 2 >> Correct


● PVC is the most environmentally damaging plastic. The PVC lifecycle -- its production,
use, and disposal -- results in the release of toxic, chlorine-based chemicals. These toxins
are building up in the water, air and food chain.
● PVC is unsustainable because it uses massive amounts of chlorine and dioxin, which
are both toxic to the environment, according to Greenpeace.
● In terms of toxicity, PVC is considered as the most hazardous plastic. The use of it may
leach a variety of toxic chemicals such as bisphenol A (BPA), phthalates, lead, dioxins,
mercury, and cadmium.

Statement 3 >> Correct


● Resin is the main component in the production of PVC. It is white, brittle solid material
available in powder form or granules.

Statement 4 >> Incorrect


● PVC is amorphous in nature and is directly related to the polar chlorine atoms in its
molecular structure. Hence it is easily combined with other chemicals/substances.

Additional information
● PVC is an excellent material for insulation due to its high dielectric strength and vapour
barrier capacity.
● It can withstand extreme climatic conditions, shock and is free from corrosion.
Hence, it is the preferred method for several outdoor applications.
● Since the durability is more, long-life is assured.
● PVC products are self-extinguishing due to high chlorine content.
● PVC can be made more flexible and softer by adding plasticizers such as phthalate and
can be bent as per requirement.
● PVC is an intrinsic flame retardant.
● It offers good tensile strength and is rigid by nature.
45
iPTS24 ENVIRONMENT ilearnoffc@gmail.com | 8089166792
● It is economical and is an affordable solution.
● It demands less maintenance and offers resistance to grease and oil.

Why this question?


Bisphenol and polycarbonate were asked before. Also, polyvinyl chloride was
one of the options. Hence, there is more probability to ask again.

79. B
Approach
● The largest sources of sulfur dioxide (SO2) emissions are generally from power plants and
industrial facilities burning fossil fuels. Statement 1 is not correct. Also FGD is indeed a
crucial technology used to reduce sulphur dioxide emission. Statement 2 is correct.

Statement 1 >> Incorrect


● According to the EPA, the largest sources of SO2 emissions are from fossil fuel
combustion at power plants (73%) and other industrial facilities (20%).
● Smaller sources of SO2 emissions include industrial processes such as extracting
metal from ore, and the burning of high sulphur containing fuels by locomotives,
large ships, and non-road equipment. Recent federal measures to limit sulphur in fuels
have reduced SO2 emissions from on-road and off-road mobile sources such as cars,
trucks and construction equipment.

Statement 2 >> Correct


● Flue Gas Desulphurisation (FGD) is the process of removing sulfur compounds from
the exhaust emissions of fossil-fueled power stations.
● This is done through the addition of absorbents, which can remove up to 95% of the
sulphur dioxide from the flue gas.
● Flue gas is the material emitted when fossil fuels such as coal, oil, natural gas, or
wood are burned for heat or power.

Additional information
● India continues to occupy the top spot among SO2 emitters. In 2019, India emitted 21%
of global anthropogenic (human-made) SO2 emissions, nearly double that of the
second-ranked global emitter, Russia.
● Short-term exposures to SO2 can harm the human respiratory system and make
breathing difficult. People with asthma, particularly children, are sensitive to these effects
of SO2.
● The WHO has estimated that it causes 4.2 million deaths globally per year.
● SO2 emissions that lead to high concentrations of SO2 in the air generally also lead
to the formation of other sulfur oxides (SOx).
● SOx can react with other compounds in the atmosphere to form small particles.
These particles contribute to Particulate Matter (PM) pollution.
● Thermal utilities which produce 75% of the country’s power, account for some 80% of
industrial emissions of sulphur and nitrous oxides, which cause lung diseases, acid rain
and smog.
● The high levels of damaging SO2 pollution in India are avoidable much sooner as FGD
systems have proved successful in reducing emission levels in China, the country
responsible for the highest level in 2005.

46
iPTS24 ENVIRONMENT ilearnoffc@gmail.com | 8089166792
Why this question?
Recently, the Ministry of Power has informed Lok Sabha that, for compliance
with Sulphur dioxide (SO2) emission norms, Thermal Power Plants are
installing Flue Gas Desulphurisation (FGD) equipment.
The Ministry in 2022 had extended the deadline for Coal-Fired power plants
to install FGD to cut sulphur emissions by two years.

80. A
Approach
● Statement 3 is correct. Use this knowledge to eliminate the wrong options.

Statement 1 >> Incorrect


● Leaded gasoline has declined considerably in recent decades, although it was a major
historical source.

Statement 2 >> Incorrect


● Blue baby syndrome, or methemoglobinemia, is often associated with exposure to nitrate
or nitrite in drinking water. These compounds can interfere with the normal oxygen-
carrying capacity of haemoglobin in the blood, leading to a bluish discoloration of the
skin in affected individuals, especially infants.

Statement 3 >> Correct


● Lead poisoning can be treated, but any damage caused cannot be reversed.

Additional information
● It is a type of poisoning that occurs when lead accumulates in the body, often over a
period of months or years.
● 23 states exceed the permissible blood lead level of 5 µg/dL margin (WHO safe limit).
● Children are particularly vulnerable to lead poisoning because their bodies are still
developing.
● India is home to a major chunk of children (275 million of the 800 million) poisoned
by lead globally.
● Most affected states:
Bihar, Uttar Pradesh, Madhya Pradesh, Jharkhand, Chhattisgarh and Andhra Pradesh.
● Deaths due to lead poisoning have continued to raise in India even though lead use in
petrol — a key source — was phased out by 2000 in the country.
Common Sources of Lead Exposure:
o Lead in water from the use of leaded pipes
o Lead from active industry, such as mining and the unsound recycling of used
lead-acid batteries (ULABs)
o Lead-based paint and pigments
o Leaded gasoline (which has declined considerably in recent decades, but was a
major historical source)
o Lead solder in food cans
o Lead in spices, cosmetics, ayurvedic medicines, toys and other consumer
products.
o Children are also exposed to lead in-utero through exposure of their mothers,
with adverse impacts on neurobehavioural development.

47
iPTS24 ENVIRONMENT ilearnoffc@gmail.com | 8089166792
Why this question?
Lead poisoning poses a hidden threat in India, with over half of children
having blood lead levels above 5 micrograms per deciliter (µg/dL) (WHO safe
level), according to studies.

81. B
Approach
● With the basic understanding, ie, ground level ozone is not directly emitted from a specific
source one can arrive at the correct answer. With respect to the topic 'Ozone', it is the key
basic fact that an aspirant should know.

Statement 1 >> Incorrect


● Ground-level ozone is not directly emitted from any specific source.

Statement 2 >> Correct


● Ground-level ozone, also known as tropospheric ozone, is a colourless and highly
irritating gas that forms near the Earth's surface, typically within two miles above the
ground.

Statement 3 >> Correct


● It is formed through complex interactions between Nitrogen Oxides (NOx), Volatile
Organic Compounds (VOCs), and carbon monoxide emitted from vehicles, power plants,
factories, and other combustion sources. These compounds undergo cyclic reactions in
the presence of sunlight to generate ground-level ozone.

Additional information
● When NOx and VOCs interact in the presence of sunlight, they undergo complex chemical
reactions that lead to the formation of ground-level ozone.
● Ground-level ozone is a significant air pollutant and can have harmful effects on human
health, vegetation, and ecosystems.
● Ozone in the air we breathe can harm our health, especially on hot sunny days when
ozone can reach unhealthy levels. People at greatest risk of harm from breathing air
containing ozone include people with asthma.
● Elevated exposures to ozone can affect sensitive vegetation and ecosystems, including
forests, parks, wildlife refuges and wilderness areas.
Why this question?
According to a new analysis by the Centre for Science and Environment
(CSE), parts of the Delhi-NCR region witnessed ground-level ozone exceeding
the national standards on 87 out of 92 days between March and May in
2023.

82. C
Approach
● The Methane Alert and Response System (MARS) was in the news very often. As per the
changed pattern of UPSC, important initiatives are relevant with respect to the
examination point of view. This is kind of a factual question.

48
iPTS24 ENVIRONMENT ilearnoffc@gmail.com | 8089166792
Statement 1 >> Correct
● It is a satellite-based system to detect methane emissions and tackle them to slow climate
change.

Statement 2 >> Correct


● The data-to-action platform was set up as part of the UN Environment Programme’s
(UNEP) International Methane Emissions Observatory (IMEO) strategy to get policy-
relevant data into the right hands for emissions mitigation.

Statement 3 >> Correct


● The Methane Alert and Response System (MARS) was launched at the 27th Conference
of Parties (COP27) to the United Nations Framework Convention on Climate Change in
Sharm El-Sheikh, Egypt.

Additional information
● It is a satellite-based system to detect methane emissions and tackle them to slow
climate change.
● The system will be the first publicly available global system to connect methane
detection to notification processes transparently.
● It will use state-of-the-art satellite data to identify significant emission events, notify
relevant stakeholders, and support and track mitigation progress.
● MARS partners will also provide technical or advisory services, such as help in assessing
mitigation opportunities.
● UNEP will monitor the event location and make the data and analysis available to the
public between 45 and 75 days after detection.
About Methane:
● Methane is a short-lived climate pollutant like hydrofluorocarbons and stays in the
Earth’s atmosphere for a few years, unlike carbon dioxide.
● Methane is the second-most abundant greenhouse gas in the atmosphere, after carbon
dioxide
● Methane is an 80 times more potent greenhouse gas than carbon dioxide at trapping
atmospheric heat in the 20 years following its release.
● Methane has contributed to about one-third of the current anthropogenic greenhouse
gas-driven warming.
Why this question?
The surge in methane levels in Earth's atmosphere has raised concerns
about the planet's ongoing climate transition. It was often in the news for
this reason.

83. C
Approach
● Radiation is an all-time relevant concept. These are mostly factual, but with the basic and
minimal understanding one can arrive at the correct answer. Least scope for elimination
and all.

Statement 1 >> Correct


● Non-ionizing radiation is a type of lower energy radiation that cannot detach electrons
from atoms or molecules, whether they are part of matter or living things.

49
iPTS24 ENVIRONMENT ilearnoffc@gmail.com | 8089166792
● Visible, infrared, and ultraviolet light, microwaves, radio waves, and radiofrequency
energy from cell phones are all examples of non-ionizing radiation.
● They have a limited ability to penetrate and have an impact on the chemicals and cells
that they absorb.

Statement 2 >> Correct


● Ionizing radiation is a type of radiation that has enough energy to separate electrons
from atoms or molecules, which results in atomic-level alterations when it interacts with
anything, including living things.
● The word “ionizing” radiation refers to changes that typically result in the formation
of ions (atoms or molecules that are Examplescally charged).
● Example: X-rays, cosmic rays, and atomic radiations are some of them (radiations emitted
by radioactive elements).

Additional information
● Radioactive pollution is the term for the dangerous level of radiation emitted by radioactive
elements.
● Radiation exposure from all man-made sources contributes 98% of the population’s
dose and accounts for 20% of the population’s overall exposure
● Radioactive Pollution is defined as the increase in the natural radiation levels in the
environment that pose a serious threat to humans and other life forms.
● Radioactive contamination is the deposition of or presence of radioactive substances on
surfaces or within solids, liquids or gases (including the human body), where their
presence is unintended or undesirable (International Atomic Energy Agency definition).
Why this question?
Radiation exposure from all man-made sources contributes 98% of the
population’s dose and accounts for 20% of the population’s overall exposure.

84. A
Approach
● The only thing you need to know is that India is a signatory to the MARPOL Convention.
Use this knowledge to eliminate the options with 3.

Statement 1 >> Incorrect


● In 2011, IMO became the first international regulator for a transport sector to adopt
globally binding energy efficiency requirements, which apply to all ships globally,
regardless of trading pattern or flag State, aimed at reducing greenhouse gas
emissions from international shipping.

Statement 2 >> Incorrect


● Annex I: Regulations for the Prevention of Pollution by Oil
● Annex IV: Prevention of Pollution by Sewage from Ships

Statement 3 >> Correct


● India has signed the International Convention for the Prevention of Pollution from Ships
(MARPOL).

50
iPTS24 ENVIRONMENT ilearnoffc@gmail.com | 8089166792
Additional information
● MARPOL is one of the most significant international marine environmental
conventions.
● The International Convention for the Prevention of Pollution from Ships (MARPOL)
is the main international convention covering prevention of pollution of the marine
environment by ships from operational or accidental causes.
● The MARPOL Convention was adopted on 2 November 1973 at IMO. The Protocol of 1978
was adopted in response to a spate of tanker accidents in 1976-1977.
● The current convention is a combination of the 1973 Convention and the 1978 Protocol,
which entered into force on 2 October 1983.
● The Convention includes regulations aimed at preventing and minimizing pollution from
ships - and currently includes six technical Annexes:
● Annex I: Regulations for the Prevention of Pollution by Oil
● Annex II: Regulations for the Control of Pollution by Noxious Liquid Substances in Bulk
● Annex III: Prevention of Pollution by Harmful Substances Carried by Sea in Packaged
Form
● Annex IV: Prevention of Pollution by Sewage from Ships
● Annex V: Prevention of Pollution by Garbage from Ships
● Annex VI: Prevention of Air Pollution from Ships.
Why this question?
MARPOL was adopted in 1973. In its 50th Anniversary year, MARPOL was
chosen as the World Maritime theme for 2023. This year's World Maritime
theme is "MARPOL at 50 – Our commitment goes on".

85. B
Approach
● It is often seen in the news as part of various discussions. Aspirants are expected to have
a clear idea on the outcome of different important conventions. With that understanding
one can easily arrive at the correct answer.

Option B >> Correct


● Prior Informed Consent (PIC) Procedure: It is one of the main provisions of the Rotterdam
Convention of 1998 that states that parties that are importing the chemicals as listed in
Annex III of the Convention have to formally obtain and disseminate the decision of
receiving future shipments of such chemicals.

Additional information
● The importing pirates have to ensure that decisions related to their import are well
in compliance by the exporting parties.
● There are 52 chemicals listed in Annex III of the Rotterdam Convention for the trade of
which parties have to follow the mechanism of Prior Informed Consent (PIC) Procedure.
● The Convention creates legally binding obligations for the implementation of the Prior
Informed Consent (PIC) procedure.
Why this question?
Iprodione & Terbufos are dangerous for the health of humans, aquatic
animals and India is among largest exporters of Terbufos.The two chemicals
are included in the PIC procedure as “pesticides” based on the decision
guidance documents approved by the chemical review committee.

51
iPTS24 ENVIRONMENT ilearnoffc@gmail.com | 8089166792
86. B
Approach
Aspirants are advised to be aware of important organisations facts related to those
organisations. To answer this question, factual knowledge is highly recommended.
● A megadiverse country is one of a group of nations that harbours the majority of Earth's
species and high numbers of endemic species. The UNEP-World Conservation Monitoring
Centre identified 17 megadiverse countries in 1998. Many of them are located at least
partially in tropical or subtropical regions.
● Australia is home to 84% of plant species, mammals, and 45% of birds. Other
countries are the United States, India, China, South Africa, Brazil, Mexico,
Madagascar, Congo, Indonesia, Malaysia, Ecuador, Philippines, Venezuela, Peru,
Colombia, and Papua New Guinea.
Thailand, Panama and Guyana are not part of megadiverse countries.

87. A
Approach
Aspirants are advised to be aware of important terms related to biodiversity.To answer
this question factual knowledge is highly recommended.

Exotic species
● Species which are not the natural inhabitants of the local habitat but are introduced into
the system.
88. B
Approach
● The question is factual in nature. Aspirants should be aware of important Ramsar sites
and their general characteristics.

Option B >> Correct


● Tso Kar Wetland Complex is a high-altitude wetland complex is found at more than
4,500 metres above sea level in the Changthang region of Ladakh. The complex includes
two connected lakes, the freshwater Startsapuk Tso and the larger hypersaline Tso
Kar; it presents a notable example of two such lakes existing in close proximity.
● Inhabiting the Site are numerous threatened species including the endangered saker
falcon (Falco cherrug) and Asiatic wild dog or dhole (Cuon alpinus laniger), and the
vulnerable snow leopard (Panthera uncia). The Site also acts as an important stopover
ground for migratory birds along the Central Asian Flyway and is one of the most
important breeding areas in India for the black-necked crane (Grus nigricollis).

89. A
Approach
● The question is factual in nature. However 500km in statement 2 is a relatively large
number given the proximity of settlements near Aravallis.

Statement 1 >> Correct


● The OECM tag is given by the International Union for Conservation of Nature (IUCN)
to areas that are not protected but support rich biodiversity. Aravalli Biodiversity Park
in Gurugram has been notified as the first OECM site in the country.
52
iPTS24 ENVIRONMENT ilearnoffc@gmail.com | 8089166792
Statement 2 >> Incorrect
● On 25th March 2023, Bhupender Yadav, Union Minister for Environment, Forest and
Climate Change, launched the Aravalli Green Wall Project, a major initiative to green the
5 km buffer area around the Aravalli Hill Range in four states (Haryana, Rajasthan,
Gujarat and Delhi) where the Aravalli hills span over 6 million hectares of land.

90. D
Approach
Aspirants are advised to be aware of important facts related to pollution. To answer this
question, factual knowledge is highly recommended.

Metal mining pollution refers to the contamination and environmental degradation


caused by the extraction and processing of Metallic Ores to obtain valuable
metals.

● Sources of Metal Mining Pollution:


■ Tailings: Tailings are finely ground rock particles left over after the valuable metals
have been extracted from the ore. These tailings often contain hazardous elements
like mercury, arsenic, lead, cadmium, and other toxic substances that can
contaminate nearby water sources and soil.
■ Acid Mine Drainage (AMD): AMD occurs when sulfide minerals in the mined rocks
are exposed to air and water, leading to the production of sulfuric acid.
○ This acid can contaminate rivers, streams, and groundwater, posing a
significant threat to aquatic life and ecosystems.
■ Airborne Pollution: Dust and particulate matter generated during mining operations
can become airborne, spreading pollutants such as heavy metals and other harmful
compounds. Inhalation of these pollutants can pose health risks to both miners and
nearby communities.
■ Chemical Usage: Chemicals such as cyanide and sulfuric acid are often used in metal
extraction processes. Accidental spills or inadequate containment of these chemicals
can result in contamination of soil and water, causing serious environmental
damage.

91. D
Approach
Aspirants are advised to be aware of important facts related to ecology. To answer this
question, factual knowledge is highly recommended.

● Ecological Footprint adds up all the biologically productive areas for which a population,
a person or a product competes. It measures the ecological assets that a given population
or product requires to produce the natural resources it consumes.
The Ecological Footprint tracks the use of productive surface areas. Typically
these areas are: cropland, grazing land, fishing grounds, built-up land, forest area,
and carbon demand on land.

53
iPTS24 ENVIRONMENT ilearnoffc@gmail.com | 8089166792
92. A
Approach
● The question is based on recent current affairs.

Option A >> Correct


The Pench Tiger Reserve (PTR) in Maharashtra has attained the distinction of
becoming India’s inaugural Dark Sky Park and the fifth in Asia. The decision to
designate Pench as the first Dark Sky Park is driven by the need to shield the night sky
from light pollution. The Dark and Quiet Skies for Science and Society Working Group,
led by the International Astronomical Union, has proposed the establishment of ‘Dark
Sky Oases’ by national and local governments.

93. D
Approach
● The question is factual in nature. However, application of some logic can help solve.
Community Reserves are geographically small entities enough to be declared as such only
by the State Governments. Hence, Statement 1 can be assumed to be Correct.

Statement 1 >> Correct


● Any community or private land may be designated by the State Government as a
Community Reserve provided the inhabitants of that community or the person concerned
consent to provide such areas for the preservation of the local flora and fauna as well as
their customs, cultures, and practises.
● The Reserve is managed by a community reserve management committee.

Statement 2 >> Correct


● These protected area categories were first introduced in the Wildlife (Protection)
Amendment Act of 2002 − the amendment to the Wildlife Protection Act of 1972.

Additional information:
● Within the Community Reserve, no changes to the way land are used are permitted
unless they are authorised by a resolution that has been approved by both the State
Government and the Management Committee.
● In India, there are 219 active community reserves, totalling 1446.28 km2, or 0.043 per
cent of the nation’s landmass.

94. D
Approach
● Relatively factual question. Aspirants are advised to know about such materials like Fly
ash, Biochar, etc.

Statement 1 >> Correct


● Biochar is fine-grained charcoal high in organic carbon that is resistant to
decomposition

Statement 2 >> Incorrect


● Created after heating of biomass in the absence of oxygen
54
iPTS24 ENVIRONMENT ilearnoffc@gmail.com | 8089166792
● A temperature of 450 degrees Celsius or more needed to convert stubble biomass to
biochar in a few hours to 24 hours’ time

Statement 3 >> Correct


● When added to the soil, increases holding capacity of water from about 11 wt% (weight
percentage) to 22 wt%.

Additional information
● Acts as a fertiliser as it can increase microbial activity and helps retention of nutrients
in the soil
● Takes over 800 years to biodegrade, so can isolate carbon and counteract global
warming.

95. A
Approach
● Some Ants are known for farming their food. This information in the description can be
helpful in narrowing down the option.

Option A >> Correct


● Ants are as old as dinosaurs. There are more than 800 different kind of ants in India.
Ants can lift 20 times their body weight! It's like a human being able to pick up a car!
Ants don't have ear they use the vibrations in the ground to hear by picking them up in
the subgenual organ which is located below the knee. Just like we raise cows, sheep,
pigs, chicken, and fish in order to obtain a food source, ants will do the same with other
insects. The most common occurrence of this is with aphids.
● Ants do not breathe like we do. They take in oxygen through tiny holes all over the body
called spiracles. They emit carbon dioxide through these same holes. The heart is a long
tube that pumps colorless blood from the head throughout the body and then back up
to the head again.

96. D
Approach
● Relatively easy question. As all the options could be employed in road construction
industry.

Statement 1, 2, 3 and 4 >> Correct


● Steel slag, a by-product of steel making, is produced during the Separation of the
molten steel from impurities in steel-making furnaces. One of the ingredients of asphalt
is steel slag, a by-product of the steel and iron production processes.

● NHAI project to evaluate the performance of Phosphor-Gypsum on a National Highway


and to generate confidence among various stakeholders on the use of Phosphor-Gypsum
waste material in Highway construction.

● Fly ash, a byproduct of thermal power plants, plays a vital role in road construction.
Fly-ash can be used in road construction as long as it meets the Indian Road Congress's
standards and specifications.
55
iPTS24 ENVIRONMENT ilearnoffc@gmail.com | 8089166792
● Bamboos are used to build road fences like the recent “Bahubali”. The 200-meter-long
bamboo crash barrier, called Bahubali, is made from Bambusa Balcooa, treated with
creosote oil, and coated with recycled High-Density Polyethylene (HDPE).

97. B
Approach
● This is a purely factual question. Attempt such questions with a clear idea about the facts.

Option B >> Correct


● The Intergovernmental Negotiating Committee (INC) was established in February
2022, at the 5th session of the United Nations Environment Assembly (UNEA-5.2).
● A historic resolution was adopted to develop an international legally binding instrument
on plastic pollution, including in the marine environment with the ambition to complete
the negotiations by the end of 2024.
● Parties will be expected to promote national action plans to work towards the prevention,
reduction and elimination of plastic pollution and to support regional and international
cooperation.
Why this question?
2024 is believed to be crucial at reaching a legally binding agreement on
plastic pollution and hence INC’s role is likely to get more attention this year.

98. C
Approach
● This is a factual question. However, Statement 2 can be assumed as a right statement
because it is logical that integrated farming helps in adaptation; eliminating Options B
and D.

Statement 1 >> Correct


● The National Adaptation Fund for Climate Change (NAFCC) is a Central Sector
Scheme which was set up in the year 2015-16
● The overall aim of NAFCC is to support concrete adaptation activities which mitigate the
adverse effects of climate change.
● The objective of the fund is to assist State and Union Territories that are particularly
vulnerable to the adverse effects of climate change in meeting the cost of adaptation.
● National Bank for Agriculture and Rural Development (NABARD) is the National
Implementing Entity (NIE).

Statement 2 >> Correct


● The projects related to adaptation in sectors such as agriculture, animal husbandry,
water, forestry, tourism etc. are eligible for funding under NAFCC.
● The fund can also be utilized for promotion of integrated farming system

56
iPTS24 ENVIRONMENT ilearnoffc@gmail.com | 8089166792
99. B
Approach
● If you think about the need for the x-ray images of wildlife products, you can reach at
Option B. Because they are useful in countering wildlife trafficking and WCCB is the major
authority in India to do that.

Option B >> Correct


● X-Ray Manual is a compilation of x-ray images of commonly smuggled wildlife
articles across India’s borders. This handbook is prepared by the Wildlife Crime Control
Bureau (WCCB).
● This exercise is an attempt to make the identification of wildlife contraband easier for
the personnel handling the scanners so that suspicious items can be detected and put
to further confirmatory scientific and forensic examination for legal action against the
defaulter.

Why this question?


WCCB appears frequently in the news for various reasons. Recently India,
Nepal and Bhutan initiated a coordinated framework for countering
wildlife trafficking in South Asia, and India is represented by WCCB in
this initiative.

100. C
Approach:
● Relatively factual question. Space debris and pollution is an important topic.

Option C >> Correct


● The Kessler Syndrome is a phenomenon in which the amount of junk in orbit around
Earth reaches a point where it just creates more and more space debris, causing big
problems for satellites, astronauts and mission planners.

57
iPTS24 ENVIRONMENT ilearnoffc@gmail.com | 8089166792

You might also like